Você está na página 1de 191

www.rejinpaul.

com
MA2111/ Engineering Mathematics-I

UNIT 1 MATRICES

CHARACTERISTIC EQUATION: The equation is called the characteristic equation of the


matrix A

Note:

1. Solving , we get n roots for and these roots are called characteristic roots or eigen values
or latent values of the matrix A
2. Corresponding to each value of , the equation AX = has a non-zero solution vector X
If be the non-zero vector satisfying AX = , when , is said to be the latent vector or eigen
vector of a matrix A corresponding to

A
CHARACTERISTIC POLYNOMIAL: The determinant when expanded will give a polynomial,
which we call as characteristic polynomial of matrix A

Working rule to find characteristic equation:


SH
YU
For a 3 x 3 matrix:
TH

Method 1:The characteristic equation is


A

Method 2:Its characteristic equation can be written as where


PR

, ,

For a 2 x 2 matrix:

Method 1:The characteristic equation is

Method 2:Its characteristic equation can be written as where


,

Problems:

1. Find the characteristic equation of the matrix ( ) [Anna University May 2003]

Get useful study materials from www.rejinpaul.com


www.rejinpaul.com
MA2111/ Engineering Mathematics-I
Solution: Let A = ( ). Its characteristic equation is

where ,

= 1(2) – 2(0) = 2

Therefore, the characteristic equation is

2. Find the characteristic equation of ( )

Solution: Its characteristic equation is , where


= 8 + 7 + 3 = 18,

| | | | | |

A
, = 8(5)+6(-10)+2(10) = 40 -60 + 20 = 0

Therefore, the characteristic equation is SH


YU
3. Find the characteristic polynomial of ( )
TH

Solution: Let A = ( )
A

The characteristic polynomial of A is where =3


PR

+ 2 = 5 and = 3(2) – 1(-1) = 7

Therefore, the characteristic polynomial is

EIGEN VALUES AND EIGEN VECTORS OF A REAL MATRIX:

Working rule to find eigen values and eigen vectors:

1. Find the characteristic equation


2. Solve the characteristic equation to get characteristic roots. They are called eigen values
3. To find the eigen vectors, solve [ ] for different values of

Note:

Get useful study materials from www.rejinpaul.com


www.rejinpaul.com
MA2111/ Engineering Mathematics-I
1. Corresponding to n distinct eigen values, we get n independent eigen vectors
2. If 2 or more eigen values are equal, it may or may not be possible to get linearly independent eigen
vectors corresponding to the repeated eigen values
3. If is a solution for an eigen value , then c is also a solution, where c is an arbitrary constant.
Thus, the eigen vector corresponding to an eigen value is not unique but may be any one of the vectors c
4. Algebraic multiplicity of an eigen value is the order of the eigen value as a root of the characteristic
polynomial (i.e., if is a double root, then algebraic multiplicity is 2)
5. Geometric multiplicity of is the number of linearly independent eigen vectors corresponding to

Non-symmetric matrix: If a square matrix A is non-symmetric, then A ≠

Note:

1. In a non-symmetric matrix, if the eigen values are non-repeated then we get a linearly independent set of

A
eigen vectors
2.
SH
In a non-symmetric matrix, if the eigen values are repeated, then it may or may not be possible to get
linearly independent eigen vectors.
YU
If we form a linearly independent set of eigen vectors, then diagonalization is possible through similarity
transformation
TH

Symmetric matrix: If a square matrix A is symmetric, then A =


A

Note:
PR

1. In a symmetric matrix, if the eigen values are non-repeated, then we get a linearly independent and pair
wise orthogonal set of eigen vectors
2. In a symmetric matrix, if the eigen values are repeated, then it may or may not be possible to get linearly
independent and pair wise orthogonal set of eigen vectors
If we form a linearly independent and pair wise orthogonal set of eigen vectors, then diagonalization is
possible through orthogonal transformation

Problems:

1. Find the eigen values and eigen vectors of the matrix ( ) [Anna University Tvli. May/June

2011]

Get useful study materials from www.rejinpaul.com


www.rejinpaul.com
MA2111/ Engineering Mathematics-I
Solution: Let A = ( ) which is a non-symmetric matrix

To find the characteristic equation:

The characteristic equation of A is where


,

= 1(-1) – 1(3) = - 4

Therefore, the characteristic equation is i.e., or

Therefore, the eigen values are 2, -2

A
To find the eigen vectors:

[ ]
SH
YU
[( ) ( )] [ ] [ ] [( ) ( )] [ ] [ ]
TH

[ ][ ] [ ] --------------- (1)
A

Case 1: If [ ][ ] [ ] [From (1)]


PR

i.e., [ ][ ] [ ]

i.e.,

i.e., we get only one equation

Therefore [ ]

Case 2: If [ ][ ] [ ] [From (1)]

Get useful study materials from www.rejinpaul.com


www.rejinpaul.com
MA2111/ Engineering Mathematics-I
i.e., [ ][ ] [ ]

i.e.,

i.e., we get only one equation

Hence, [ ]

2. Find the eigen values and eigen vectors of [ ] [Anna University Tvli. May/June 2011]

A
Solution: Let A = [ SH
] which is a non-symmetric matrix
YU
To find the characteristic equation:
TH

Its characteristic equation can be written as where


A

,
PR

| | | | | |

= 2 (-5)-2 (-6)-7(2) = -10 + 12 – 14 = -12

Therefore, the characteristic equation of A is

3 -4 0

√ √

Get useful study materials from www.rejinpaul.com


www.rejinpaul.com
MA2111/ Engineering Mathematics-I
Therefore, the eigen values are 3, 1, and -4

A is a non-symmetric matrix with non-repeated eigen values

To find the eigen vectors:

[ ]

[ ][ ] [ ]

Case 1: If [ ][ ] [ ]

A
i.e., [ ][ ] [ ]

--------- (1)
SH
YU
------- (2)
TH

-------- (3)
A

Considering equations (1) and (2) and using method of cross-multiplication, we get,
PR

2 -7 1 2

0 2 2 0

Therefore, [ ]

Case 2: If ,[ ][ ] [ ]

Get useful study materials from www.rejinpaul.com


www.rejinpaul.com
MA2111/ Engineering Mathematics-I
i.e., [ ][ ] [ ]

-------- (1)

-------- (2)

-------- (3)

Considering equations (1) and (2) and using method of cross-multiplication, we get,

2 -7 -1 2

A
-2 2 2 -2

SH
YU
Therefore, [ ]
A TH

Case 3: If [ ][ ] [ ]
PR

-------- (1)

-------- (2)

-------- (3)

Considering equations (1) and (2) and using method of cross-multiplication, we get,

2 -7 6 2

5 2 2 5

Get useful study materials from www.rejinpaul.com


www.rejinpaul.com
MA2111/ Engineering Mathematics-I

Therefore, [ ]

3. Find the eigen values and eigen vectors of [ ] [Anna University November/December

2010, May/June 2010], [Anna University CBT January 2011]

Solution: Let A = [ ] which is a non-symmetric matrix

A
To find the characteristic equation:

Its characteristic equation can be written as


SH where
YU
,

| | | | | |
TH

,
A

= 2(4)-2(1)+1(-1) = 5
PR

Therefore, the characteristic equation of A is


1

1 -6 5 0
√ √

Therefore, the eigen values are 1, 1, and 5

A is a non-symmetric matrix with repeated eigen values

To find the eigen vectors:

[ ]
8

Get useful study materials from www.rejinpaul.com


www.rejinpaul.com
MA2111/ Engineering Mathematics-I

[ ][ ] [ ]

Case 1: If [ ][ ] [ ]

i.e., [ ][ ] [ ]

--------- (1)

------------- (2)

A
------------ (3)

SH
Considering equations (1) and (2) and using method of cross-multiplication, we get,
YU

2 1 -3 2
TH

-2 1 1 -2
A
PR

Therefore, [ ]

Case 2: If ,[ ][ ] [ ]

i.e., [ ][ ] [ ]

Get useful study materials from www.rejinpaul.com


www.rejinpaul.com
MA2111/ Engineering Mathematics-I

All the three equations are one and the same. Therefore,

Put

Therefore, [ ]

Put

Therefore, [ ]

A
4. Find the eigen values and eigen vectors of [ ][Anna University Madurai, January 2011]

SH
YU
Solution: Let A = [ ] which is a non-symmetric matrix
TH

To find the characteristic equation:

Its characteristic equation can be written as where


A

,
PR

| | | | | |

,
= 2(-4)+2(-2)+2(2) = - 8 – 4 + 4 = - 8
Therefore, the characteristic equation of A is

1 0 -4 0

Therefore, the eigen values are 2, 2, and -2

A is a non-symmetric matrix with repeated eigen values


10

Get useful study materials from www.rejinpaul.com


www.rejinpaul.com
MA2111/ Engineering Mathematics-I
To find the eigen vectors:

[ ]

[ ][ ] [ ]

Case 1: If [ ][ ] [ ]

i.e., [ ][ ] [ ]

--------- (1)

A
------------- (2)
SH
YU
------------ (3) . Equations (2) and (3) are one and the same.

Considering equations (1) and (2) and using method of cross-multiplication, we get,
A TH

-1 1 2 -1
PR

3 1 1 3

Therefore, [ ]

Case 2: If ,[ ][ ] [ ]

i.e., [ ][ ] [ ]

11

Get useful study materials from www.rejinpaul.com


www.rejinpaul.com
MA2111/ Engineering Mathematics-I
---------- (1)

---------------- (2)

------------ (3)

Considering equations (1) and (2) and using method of cross-multiplication, we get,

-2 2 0 -2

-1 1 1 -1

A
Therefore, [ ]
SH
YU
We get one eigen vector corresponding to the repeated root
TH

5. Find the eigen values and eigen vectors of [ ] [Anna University Tvli., January 2010,2011]
A
PR

Solution: Let A =[ ] which is a symmetric matrix

To find the characteristic equation:

Its characteristic equation can be written as where

| | | | | |

,
= 1(4)-1(-2)+3(-14) = - 4 + 2 -42 = - 36

Therefore, the characteristic equation of A is

12

Get useful study materials from www.rejinpaul.com


www.rejinpaul.com
MA2111/ Engineering Mathematics-I

1 -9 18 0
√ √

Therefore, the eigen values are -2, 3, and 6

A is a symmetric matrix with non- repeated eigen values

To find the eigen vectors:

[ ]

A
[ ][ ] [ ]

[
SH
][ ] [ ]
YU
Case 1: If
TH

i.e., [ ][ ] [ ]
A
PR

--------- (1)

------------- (2)

------------ (3)

Considering equations (1) and (2) and using method of cross-multiplication, we get,

1 3 3 1

7 1 1 7

13

Get useful study materials from www.rejinpaul.com


www.rejinpaul.com
MA2111/ Engineering Mathematics-I
Therefore, [ ]

Case 2: If ,[ ][ ] [ ]

i.e., [ ][ ] [ ]

---------- (1)

---------------- (2)

3 ------------ (3)

A
SH
Considering equations (1) and (2) and using method of cross-multiplication, we get,
YU
1 3 -2 1
TH

2 1 1 2
A
PR

Therefore, [ ]

Case 3: If ,[ ][ ] [ ]

i.e., [ ][ ] [ ]

---------- (1)

---------------- (2)

14

Get useful study materials from www.rejinpaul.com


www.rejinpaul.com
MA2111/ Engineering Mathematics-I
3 ------------ (3)

Considering equations (1) and (2) and using method of cross-multiplication, we get,

1 3 -5 1

-1 1 1 -1

Therefore, [ ]

A
6. Find the eigen values and eigen vectors of the matrix[
SH ]. Determine the algebraic and
YU
geometric multiplicity

Solution: Let A = [ ] which is a symmetric matrix


A TH

To find the characteristic equation:


PR

Its characteristic equation can be written as where


,

| | | | | |

,
= 0 -1(-1)+ 1(1) = 0 + 1 + 1 = 2

Therefore, the characteristic equation of A is

1 -1 -2 0
√ √

15

Get useful study materials from www.rejinpaul.com


www.rejinpaul.com
MA2111/ Engineering Mathematics-I
Therefore, the eigen values are 2, -1, and -1

A is a symmetric matrix with repeated eigen values. The algebraic multiplicity of is 2

To find the eigen vectors:

[ ]

[ ][ ] [ ]

Case 1: If [ ][ ] [ ]

A
i.e., [ ][ ] [ ]

--------- (1)
SH
YU
------------- (2)
TH

------------ (3)
A

Considering equations (1) and (2) and using method of cross-multiplication, we get,
PR

1 1 -2 1

-2 1 1 -2

Therefore, [ ]

16

Get useful study materials from www.rejinpaul.com


www.rejinpaul.com
MA2111/ Engineering Mathematics-I

Case 2: If ,[ ][ ] [ ]

i.e., [ ][ ] [ ]

---------- (1)

---------------- (2)

------------ (3). All the three equations are one and the same.

Therefore, . Put

A
SH
Therefore, [ ]
YU
Since the given matrix is symmetric and the eigen values are repeated, let [ ]. is orthogonal to
A TH

[ ][ ] ------------ (1)
PR

[ ][ ] -------- (2)

Solving (1) and (2) by method of cross-multiplication, we get,

l m

1 1 1 1

1 -1 0 1

17

Get useful study materials from www.rejinpaul.com


www.rejinpaul.com
MA2111/ Engineering Mathematics-I
. Therefore, [ ]

Thus, for the repeated eigen value there corresponds two linearly independent eigen vectors
. So, the geometric multiplicity of eigen value is 2

PROPERTIES OF EIGEN VALUES AND EIGEN VECTORS:

Property 1:

(i) The sum of the eigen values of a matrix is the sum of the elements of the principal diagonal (or)
The sum of the eigen values of a matrix is equal to the trace of the matrix
(ii) Product of the eigen values is equal to the determinant of the matrix

A
Proof: Let A be a square matrix of order n. The characteristic equation of A is i.e.,

SH
--------- (1) where
YU

A TH

We know that the roots of the characteristic equation are called eigen values of the given matrix. Solving (1),
PR

we get n roots. Let the n roots be i.e., are the eigen values of A.

Already, we know that,


----------- (2)

Sum of the roots = by (1) and (2)

i.e.,

i.e.,

i.e., Sum of the eigen values = Sum of the main diagonal elements

Product of the roots = by (1) and (2)

18

Get useful study materials from www.rejinpaul.com


www.rejinpaul.com
MA2111/ Engineering Mathematics-I

i.e., Product of the eigen values = |A|

Property 2: A square matrix A and its transpose have the same eigen values (or) A square matrix A
and its transpose have the same characteristic values

Proof: Let A be a square matrix of order n. The characteristic equation of A and are -------
(1) and --------- (2)

Since the determinant value is unaltered by the interchange of rows and columns, we have |A| = |

Hence, (1) and (2) are identical.

A
Therefore, the eigen values of A and are the same

Note:
SH
YU
A determinant remains unchanged when rows are changed into columns and columns into rows

Property 3: The characteristic roots of a triangular matrix are just the diagonal elements of the matrix
TH

(or) the eigen values of a triangular matrix are just the diagonal elements of the matrix
[Anna University Madurai, January 2011]
A
PR

Proof: Let us consider the triangular matrix A = [ ].

The characteristic equation of A is

i.e., | |

On expansion, it gives,

i.e., which are the diagonal elements of matrix A

19

Get useful study materials from www.rejinpaul.com


www.rejinpaul.com
MA2111/ Engineering Mathematics-I
Property 4: If is an eigen value of a matrix A, then , is the eigen value of (or) If is an

eigen value of a matrix A, what can you say about the eigen value of matrix Prove your statement
[Anna University March 1996, Anna University May 1997]

Proof: If X be the eigen vector corresponding to then AX = --------- (1)

Pre multiplying both sides by , we get,

Dividing by we get,

A
SH
This being of the same form as (1), shows that is the eigen value of the inverse matrix
YU

Property 5: If is an eigen value of an orthogonal matrix, then is also its eigen value
TH

Proof: Definition of orthogonal matrix:


A

A square matrix A is said to be orthogonal if i.e.,


PR

Let A be an orthogonal matrix. Given is an eigen value of A.

is an eigen value of . Since , is an eigen value of . But, the matrices A and have the

same eigen values, since the determinant are the same

Hence, is also an eigen value of A

Property 6: If are the eigen values of a matrix A, then has the eigen values
(m being a positive integer)

Proof: Let be the eigen value of A and the corresponding eigen vector.

20

Get useful study materials from www.rejinpaul.com


www.rejinpaul.com
MA2111/ Engineering Mathematics-I
Then, ------- (1)

We have,

Similarly, . In general, -------- (2)

(1) and (2) are of the same form.

Hence, is an eigen value of . The corresponding eigen vector is the same .

Property 7: The eigen values of a real symmetric matrix are real numbers
[Anna University November 2001]

Proof: Let be an eigen value (may be complex) of the real symmetric matrix A. Let the corresponding eigen
vector be X.

A
Let be the transpose of A.
SH
YU
We have, AX =

Pre multiplying this equation by 1 x n matrix ̅̅̅where the bar denotes that all elements of ̅̅̅ are the complex
TH

conjugate of , we get, ̅̅̅̅AX = ̅ X ------------- (1)

̅̅
A

Taking the conjugate complex of this, we get, ̅ ̅ or ̅= ̅ ̅ since ̅ = A if A is real


PR

Taking the transpose on both sides, we get, ̅ ̅ ̅ i.e., ̅ ̅̅ i.e.,

̅̅̅ ̅ ̅̅̅ since if A is symmetric. But, from (1), ̅̅̅AX = ̅̅̅X.

Hence, ̅̅̅X = ̅ ̅ . Since̅̅̅̅ is a 1 x 1 matrix whose only element is a positive value, ̅ i.e.,

Property 8: The eigen vectors corresponding to distinct eigen values of a real symmetric matrix are
orthogonal [Anna University, November 2002]

Proof: For a real symmetric matrix A, the eigen values are real. Let be the eigen vectors to two distinct
eigen values [ are real]

21

Get useful study materials from www.rejinpaul.com


www.rejinpaul.com
MA2111/ Engineering Mathematics-I
------------- (1)

------------- (2)

Pre multiplying (1) by , we get, =

Pre multiplying (2) by , we get, ------------- (3)

But i.e., -------- (4) [Since ]

From (3) and (4),

i.e., . Since are orthogonal

Property 9: Similar matrices have same eigen values [Anna University, MIT]

A
Proof: Let A and B be two similar matrices. Then there exists a non-singular matrix P such that

B– SH
YU
=
TH

|B–
A

|=|
PR

= = |A

= |A

Therefore, A, B have the same characteristic polynomial and hence characteristic roots.

Therefore, they have the same eigen values.

Property 10: If a real symmetric matrix of order 2 has equal eigen values, then the matrix is a scalar
matrix

Proof:

Rule 1: A real symmetric matrix of order n can always be diagonalized

22

Get useful study materials from www.rejinpaul.com


www.rejinpaul.com
MA2111/ Engineering Mathematics-I
Rule 2: If any diagonalized matrix has its diagonal elements equal, then the matrix is a scalar matrix

Given: A real symmetric matrix A of order 2 has equal eigen values

By rule 1, A can always be diagonalized. Let be their eigen values, then, we get the diagonalized

matrix = [ ]. Given, . Therefore, we get [ ]. By rule 2, the given matrix is a scalar matrix.

Property 11: The eigen vector X of a matrix A is not unique.

Proof: Let be the eigen value of A. Then, there is a corresponding eigen vector X such that AX =

Multiply both sides by a non-zero scalar K.

K(AX) = K ( i.e., an eigen vector is determined by a multiplicative scalar.

A
i.e., eigen vector is not unique

Property 12: If SH
be distinct eigen values of a n x n matrix, then the corresponding
YU
eigen vectors form a linearly independent set

Property 13: If two or more eigen values are equal, it may or may not be possible to get linearly
TH

independent eigen vectors corresponding to the equal roots


A

Property 14: Two eigen vectors are called orthogonal vectors if


PR

Property 15: Eigen vectors of a symmetric matrix corresponding to different eigen values are
orthogonal

Property 16: If A and B are n x n matrices and B is a non-singular matrix then A and have
same eigen values

Problems:

1. Find the sum and product of the eigen values of the matrix [ ] [AU, March 1996]

Solution: Sum of the eigen values = Sum of the main diagonal elements = -3

23

Get useful study materials from www.rejinpaul.com


www.rejinpaul.com
MA2111/ Engineering Mathematics-I
Product of the eigen values = │A│ = -1 (1 – 1) -1(-1 – 1) + 1(1- (-1)) = 2 + 2 = 4

2. Find the sum and product of the eigen values of the matrix [ ] [AU, Dec 1999]

Solution: Sum of the eigen values = Sum of the main diagonal elements = -1

Product of the eigen values = │A│= -2(0 – 12) – 2(0 – 6) -3 (-4+1) = 24+12+9 = 45

3. The product of two eigen values of the matrix A = [ ] is 16. Find the third eigen value

[AU, April/May 2003], [AU, Tvli. April/May 2009]

Solution: Let the eigen values of the matrix be .

A
Given

We know that SH
(Since product of the eigen values is equal to the determinant of the matrix)
YU

| | = 6(9-1)+2(-6+2) +2(2-6) = 48-8-8 = 32


A TH
PR

4. Find the sum and product of the eigen values of the matrix A = ( ) without finding the roots

of the characteristic equation

Solution: We know that the sum of the eigen values = Trace of A = a + d

Product of the eigen values = │A│ = ad – bc

5. Two of the eigen values of [ ] are 2 and 8. Find the third eigen value

Solution: We know that sum of the eigen values = Sum of the main diagonal elements = 6+3+3 = 12

Given

24

Get useful study materials from www.rejinpaul.com


www.rejinpaul.com
MA2111/ Engineering Mathematics-I
Therefore, = 12

Therefore, the third eigen value = 2

6. If 3 and 15 are the two eigen values of A = [ ], find │A│, without expanding the

determinant

Solution: Given

We know that sum of the eigen values = Sum of the main diagonal elements

A
SH
YU
TH

7. If 2, 2, 3 are the eigen values of A = [ ], find the eigen values of


A
PR

Solution: By the property “A square matrix A and its transpose have the same eigen values”, the eigen
values of

8. Find the eigen values of A = [ ] without using the characteristic equation idea

Solution: Given A = [ ] . Clearly, A is an upper triangular matrix. Then, by the property “The

characteristic roots of a triangular matrix are just the diagonal elements of the matrix”, the eigen values of A
are 2, 2, 2

25

Get useful study materials from www.rejinpaul.com


www.rejinpaul.com
MA2111/ Engineering Mathematics-I
9. Find the eigen values of A = [ ]

Solution: Given A = [ ] .Clearly, A is a lower triangular matrix. Hence, by the property “the

characteristic roots of a triangular matrix are just the diagonal elements of the matrix”, the eigen values of A
are 2, 3, 4

10. Two of the eigen values of A = [ ] are 3 and 6. Find the eigen values of

Solution: Sum of the eigen values = Sum of the main diagonal elements = 3 +5+3 = 11

Given 3,6 are two eigen values of A. Let the third eigen value be k.

A
Then, 3 + 6 + k = 11
SH
YU
Therefore, the eigen values of A are 3, 6, 2

By the property “If the eigen values of A are , then the eigen values of are ”, the eigen
TH

values of are
A

11. Find the eigen values of the matrix[ ]. Hence, form the matrix whose eigen values are
PR

[AU, May 2001 P.T]

Solution: Let A =[ ]. The characteristic equation of the given matrix is where

and


Therefore, the characteristic equation is =

Therefore, the eigen values of A are 6, -1

Hence, the matrix whose eigen values are is

26

Get useful study materials from www.rejinpaul.com


www.rejinpaul.com
MA2111/ Engineering Mathematics-I

│A│ = 4 -10 = - 6; adj A = [ ]

Therefore, [ ]

12. Two eigen values of the matrix A = [ ] are equal to 1 each. Find the eigen values of

[AU, Nov/Dec 2002]

Solution: Given A = [ ]

A
Let the eigen values of A be
SH
YU
We know that, Sum of the eigen values = Sum of the main diagonal elements = 7

Therefore,
TH

Hence, the eigen values of A are 1, 1, 5


A
PR

The eigen values of are 1, 1,

13. Find the eigen values of the inverse of the matrix A = [ ]

Solution: We know that A is an upper triangular matrix. Therefore, the eigen values of A are 2, 3, 4. Hence,
by using the property “If the eigen values of A are , then the eigen values of are ”, the

eigen values of

14. Find the eigen values of given A = [ ] [AU, Oct/Nov 1997]

27

Get useful study materials from www.rejinpaul.com


www.rejinpaul.com
MA2111/ Engineering Mathematics-I
Solution: Given A = [ ]. A is an upper triangular matrix. Hence, the eigen values of A are 1, 2, 3

Therefore, the eigen values of are i.e., 1,8,27

15. If 1 and 2 are the eigen values of a 2 x 2 matrix A, what are the eigen values of ?[AU,
Model paper]

Solution: Given 1 and 2 are the eigen values of A.

Therefore, i.e., 1 and 4 are the eigen values of and 1 and are the eigen values of

16. Show that the eigen values of the real symmetric matrix A = [ ] are real

A
Solution: Given A is a real symmetric matrix.

The characteristic equation of A is where SH


YU
;


TH

Therefore, the characteristic equation of A is

Therefore, the eigen values are -3, 2 (real)


A
PR

17. If 1,1,5 are the eigen values of A = [ ], find the eigen values of 5A

Solution: By the property “If are the eigen values of A, then are the eigen values of
kA, the eigen values of 5A are 5(1), 5(1), 5(5) ie., 5,5,25

18. Find the eigen values of A, if A = [ ]

Solution: Given A = [ ]. A is an upper triangular matrix. Hence, the eigen values of A are 2, 5

The eigen values of are i.e., 4, 25

The eigen values of are i.e., 8, 125


28

Get useful study materials from www.rejinpaul.com


www.rejinpaul.com
MA2111/ Engineering Mathematics-I
The eigen values of are i.e., 16, 625

The eigen values of 3A are 3(2), 3(5) i.e., 6, 15

The eigen values of are

A–I=[ ]-[ ]=[ ]

Since A - I is an upper triangular matrix, the eigen values of A- I are its main diagonal elements i.e., 1,4

Eigen values of where

First eigen value =

A
= 3(2)3+ 5(2)2 - 6(2) + 2 = 24 + 20 -12 + 2=34

Second eigen value = SH


YU
= 3(5)3 + 5(5)2 - 6(5) + 2
TH

= 375+ 125-30 + 2 = 472


A

19. If are the eigen values of [ ], form the matrix whose eigen values are
PR

[AU, May 2001 P.T]

Solution: Let A = [ ] and let be the eigen values of A

[ ][ ] [ ]

[ ][ ] [ ]

Hence the required matrix is

[ ]

29

Get useful study materials from www.rejinpaul.com


www.rejinpaul.com
MA2111/ Engineering Mathematics-I
20. Form the matrix whose eigen values are where are the eigen values of A

=[ ] [AU, May 1998]

Solution: The matrix A – KI has the eigen values

Hence, the required matrix is A – 5I = [ ] [ ]=[ ]

21. The eigen vectors of a 3 x 3 real symmetric matrix A corresponding to the eigen values 2,3,6 are
[ ] ,[ ] and [ ] respectively. Find the matrix A [AU, Nov 2002,
April/May 2011]

Solution: Given eigen values of A are 2,3,6

A
Also given, eigen vectors are [ ], [ ] [ ]
SH
YU
√ √ √ √ √
TH

The Normalized matrix N = √ √


; = √ √ √

[√ √ √ ] [√ √ √ ]
A
PR

D=

ND = N

ND = (N

N is an orthogonal matrix. Hence N

Therefore, A = ND

√ √ √ √ √ √ √ √ √ √
A= √ √
[ ] √ √ √
= √ √
[√ √ √ ]
√ √ √
[√ √ √ ] [√ √ √ ] [√ √ √ ]

30

Get useful study materials from www.rejinpaul.com


www.rejinpaul.com
MA2111/ Engineering Mathematics-I
=[ ]=[ ]

22. Find the eigen values of adj A if A = [ ]

Solution: Given A =[ ]. A is an upper triangular matrix. Hence, the eigen values of A are 3, 4, 1

We know that

Adj A = │A│

A
The eigen values of are

│A│=Product of the eigen values = 12


SH
YU
Therefore, the eigen values of adj A is equal to the eigen values of 12 i.e., i.e., 4, 3, 12
TH

Note: A =[ ] [ ] [ ]. Here, A is an upper triangular matrix, B is a lower


A

triangular matrix and C is a diagonal matrix. In all the cases, the elements in the main diagonal are the eigen
PR

values. Hence, the eigen values of A, B and C are 1, 4, 6

23. Two eigen values of A = [ ] are equal and they are times the third. Find them

Solution: Let the third eigen value be

We know that 2+3+2 = 7

Given =

31

Get useful study materials from www.rejinpaul.com


www.rejinpaul.com
MA2111/ Engineering Mathematics-I
[ ]

Therefore, and hence the eigen values of A are 1,1, 5

24. If 2, 3 are the eigen values of [ ]

Solution: Let A =[ ]. Let the eigen values of A be 2, 3, k

We know that the sum of the eigen values = sum of the main diagonal elements

Therefore, 2 +3 +k = 2+ 2+2 = 6

A
We know that product of the eigen values = │A│

2(3)(k) = │A│
SH
YU
| |
TH

25. Prove that the eigen vectors of the real symmetric matrix A = [ ] are orthogonal in pairs
A
PR

Solution: The characteristic equation of A is


where ;

| |

The characteristic equation of A is


3 1 -7 0 36
0 3 -12 -36
1 -4 -12 0

32

Get useful study materials from www.rejinpaul.com


www.rejinpaul.com
MA2111/ Engineering Mathematics-I

Therefore,

Therefore, the eigen values of A are -2, 3, 6

To find the eigen vectors:

Case 1: When [ ][ ] [ ]

-------- (1)

---------- (2)

A
--------- (3)

Solving (1) and (2) by rule of cross-multiplication, we get, SH


YU
TH

1 3 3 1
A

7 1 1 7
PR

[ ]

Case 2: When [ ][ ] [ ]

--------- (1)

--------- (2)

--------- (3)

Solving (1) and (2) by rule of cross-multiplication, we get,

33

Get useful study materials from www.rejinpaul.com


www.rejinpaul.com
MA2111/ Engineering Mathematics-I

1 3 -2 1

2 1 1 2

[ ]

Case 3: When [ ][ ] [ ]

--------- (1)

--------- (2)

A
--------- (3)
SH
YU
Solving (1) and (2) by rule of cross-multiplication, we get,
TH

1 3 -5 1
A

-1 1 1 -1
PR

[ ]

Therefore, [ ], [ ] [ ]

To prove that:

[ ][ ]

34

Get useful study materials from www.rejinpaul.com


www.rejinpaul.com
MA2111/ Engineering Mathematics-I

[ ][ ]

[ ][ ]

Hence, the eigen vectors are orthogonal in pairs

26. Find the sum and product of all the eigen values of the matrix A = [ ].Is the matrix

singular? [AU, Oct/Nov 1997]

Solution: Sum of the eigen values = Sum of the main diagonal elements =Trace of the matrix

A
SH
Therefore, the sum of the eigen values = 1+2+7=10

Product of the eigen values = │A│ = 1(14 - 8) -2(14 - 4) + 3(4 - 2) = 6-20+ 6= - 8


YU
│A│≠0. Hence the matrix is non-singular.
TH

27. Find the product of the eigen values of A = [ ] [AU CBT J/J 2010]
A
PR

Solution: Product of the eigen values of A = │A│= | |

CAYLEY-HAMILTON THEOREM: [AU CBT J/F 2010, A/M 2011]

Statement: Every square matrix satisfies its own characteristic equation

Uses of Cayley-Hamilton theorem:

(1) To calculate the positive integral powers of A


(2) To calculate the inverse of a square matrix A

Problems:
35

Get useful study materials from www.rejinpaul.com


www.rejinpaul.com
MA2111/ Engineering Mathematics-I
1. Show that the matrix [ ] satisfies its own characteristic equation [AU, May 2001 P.T]

Solution: Let A = [ ]. The characteristic equation of A is where

The characteristic equation is

To prove

[ ][ ]=[ ]

A
[ ] [ ] [ ] [ ]

Therefore, the given matrix satisfies its own characteristic equation SH


YU
2. If A = [ ] [AU,
TH

Model paper]

Solution: Cayley-Hamilton theorem states that every square matrix satisfies its own characteristic equation.
A
PR

The characteristic equation of A is where

Therefore, the characteristic equation is

By Cayley-Hamilton theorem,

i.e.,

36

Get useful study materials from www.rejinpaul.com


www.rejinpaul.com
MA2111/ Engineering Mathematics-I
3. Verify Cayley-Hamilton theorem, find when A = [ ] [AU A/M 2002, Jan

2009, CBT Jan/Feb 2009] [AU Tvli. Jan 2010] [AU Jan 2010] [AU Madurai June 2011] [AU May/June
2010]

Solution: The characteristic equation of A is where

Therefore, the characteristic equation is

A
To prove that: ------------- (1)

[ ][ ] [
SH
]
YU
TH

[ ][ ] [ ]
A
PR

[ ] [ ] [ ] [ ]

[ ]

To find :

------------- (2)

Multiply by A on both sides,

Therefore,

37

Get useful study materials from www.rejinpaul.com


www.rejinpaul.com
MA2111/ Engineering Mathematics-I
Hence, [ ] [ ] [ ]

[ ] [ ] [ ] [ ]

To find

Multiplying (1) by

[ ] [ ] [ ]

A
[ ] [ ] [ ]
SH[ ]
YU
[ ]
TH

4. Verify that A = [ ] satisfies its own characteristic equation and hence find [AU
A

April/May 2003]
PR

Solution: Given A =[ ]. The characteristic equation of A is where

Therefore, the characteristic equation is i.e.,

To prove: ---------- (1)

[ ][ ] [ ] [ ]

38

Get useful study materials from www.rejinpaul.com


www.rejinpaul.com
MA2111/ Engineering Mathematics-I

[ ] [ ] [ ] [ ] [ ]

To find

From (1), we get,

Multiplying by on both sides, we get, [ ] [ ]

5. Find if A = [ ], using Cayley-Hamilton theorem [AU, Nov 2002]

Solution: The characteristic equation of A is where

A
SH
YU
TH

The characteristic equation of A is


A
PR

By Cayley- Hamilton theorem, ------------ (1)

To find

Multiplying (1) by we get,

------------- (2)

[ ][ ] [ ] [ ]

[ ] [ ] [ ] [ ]

39

Get useful study materials from www.rejinpaul.com


www.rejinpaul.com
MA2111/ Engineering Mathematics-I
From (2), [ ]

6. If A = [ ]

Solution: The characteristic equation of A is where


The characteristic equation of A is i.e.,

To find

A
When is divided by , let the quotient be and the remainder be

---------- (1) SH
YU
When When
TH

------------ (2)
A

------------ (3)
PR

Solving (2) and (3), we get, (2) - (3)

(2) – 2 x (3)

i.e.,

Since by Cayley-Hamilton theorem, (1)

[ ] [ ][ ]

40

Get useful study materials from www.rejinpaul.com


www.rejinpaul.com
MA2111/ Engineering Mathematics-I
7. Use Cayley-Hamilton theorem to find the value of the matrix given by (i)
(ii)

if the matrix A = [ ] [AU M/J 2009], [AU Tvli. Jan 2010]

Solution: The characteristic equation of A is where

A
The characteristic equation is

By Cayley-Hamilton theorem, we get,


SH
----------- (1)
YU
Let f(A) =
TH

Let g(A) =
A

(i)
PR

(-)

(-)

f(A) = ( )

= ------------ (2) (by (1))

Now, [ ][ ] [ ]

41

Get useful study materials from www.rejinpaul.com


www.rejinpaul.com
MA2111/ Engineering Mathematics-I
[ ] [ ] [ ] [ ]

(ii)

(-)

(-)

(-)

A
SH
g(A) = (
=0+
YU
=

= [ ] [ ] [ ]
TH

g(A) = [ ]
A
PR

8. Use Cayley-Hamilton theorem for the matrix [ ] to express as a linear polynomial in A (i)

(ii) [AU M/J 2009]

Solution: Given A = [ ]. The characteristic equation of A is where

The characteristic equation is

By Cayley-Hamilton theorem, we get, ------------ (1)

(i)
42

Get useful study materials from www.rejinpaul.com


www.rejinpaul.com
MA2111/ Engineering Mathematics-I

(-)

(by (1)) which is a linear polynomial in A

A
(ii)

SH
YU
(-)
TH

= = 0 + A + 2I = A +2I (by (1)) which is a linear


polynomial in A
A
PR

9. Using Cayley-Hamilton theorem, find when A = [ ] [AU May/June 2005]

Solution: The characteristic equation of A is where

The characteristic equation is


43

Get useful study materials from www.rejinpaul.com


www.rejinpaul.com
MA2111/ Engineering Mathematics-I
By Cayley-Hamilton theorem,

Pre-multiplying by we get,

[ ][ ] [ ] [ ]

[ ]; [ ] [ ]

([ ] [ ] [ ]) [ ]

A
10. Verify Cayley-Hamilton theorem for the matrix [ ] [AU Tvli. Jan 2009]

Solution: Given A = [ ]
SH
YU
TH

The Characteristic equation of A is where

Sum of the main diagonal elements = 1+2+1 = 4


A
PR

The characteristic equation is

To prove that:

[ ][ ] [ ] [ ]

44

Get useful study materials from www.rejinpaul.com


www.rejinpaul.com
MA2111/ Engineering Mathematics-I

[ ][ ] [ ]

=[ ]

[ ] [ ] [ ] [ ]

=[ ] [ ] [ ] [ ]

A
=[ ]

Therefore, Cayley-Hamilton theorem is verified. SH


YU
11. Verify Cayley-Hamilton theorem for the matrix (i) A = [ ] [ ] [AU CBT Jan 2011]
TH

Solution: (i) Given A = [ ]


A

The characteristic equation of A is where


PR

The characteristic equation is

To prove that:

[ ][ ] [ ] [ ]

[ ] [ ]

45

Get useful study materials from www.rejinpaul.com


www.rejinpaul.com
MA2111/ Engineering Mathematics-I

[ ] [ ]

[ ] [ ] [ ] [ ]

Hence Cayley-Hamilton theorem is verified.

(ii) Given A = [ ]

The characteristic equation of A is where

A
SH
The characteristic equation is

To prove that:
YU

[ ][ ] [ ] [ ]
TH

[ ] [ ]; [ ] [ ]
A
PR

[ ] [ ] [ ] [ ]

Hence Cayley-Hamilton theorem is verified.

12. Find using Cayley-Hamilton theorem when A =[ ] .

Solution: Given A = [ ]

The characteristic equation of A is where

46

Get useful study materials from www.rejinpaul.com


www.rejinpaul.com
MA2111/ Engineering Mathematics-I
√ √
The characteristic equation is i.e., 6, 2

By Cayley-Hamilton theorem,

To find

When is divided by , let the quotient be and the remainder be

---------- (1)

When When

------- (1)

------- (2)

A
(1) – (2)
SH
YU
Substituting in (1), we get, ( )
TH

4b =
A

Therefore, ,b=
PR

Since [ ] [ ]

( )
[ ] [ ] ( )[ ] ( )[ ] [ ]

[ ] [ ] [ ] [ ]

Therefore, [ ]

13. Find using Cayley-Hamilton theorem when A = [ ]. Also find .

Solution: Given A = [ ]

47

Get useful study materials from www.rejinpaul.com


www.rejinpaul.com
MA2111/ Engineering Mathematics-I
The characteristic equation of A is where

√ √
The characteristic equation is i.e.,

By Cayley-Hamilton theorem,

To find

When is divided by , let the quotient be and the remainder be

---------- (1)

A
When

------- (1)
When
SH
YU
------- (2)
TH

(1) – (2)
A

( )
PR

Substituting in (1), we get,

5b = 9

Therefore,

Since [ ] [ ]

( )[ ] ( )[ ] ( )[ ] [ ]

[ ] [ ]

48

Get useful study materials from www.rejinpaul.com


www.rejinpaul.com
MA2111/ Engineering Mathematics-I
=[ ] [ ] [ ]

ORTHOGONAL TRANSFORMATION OF A SYMMETRIC MATRIX TO DIAGONAL FORM:

Orthogonal matrices:

A square matrix A (with real elements) is said to be orthogonal if or

Problems:

1. Check whether the matrix B is orthogonal. Justify. B = [ ]

Solution: Condition for orthogonality is

A
To prove that:

B=[ ]; [ ]
SH
YU
TH

[ ][ ]
A

[ ] [ ]
PR

Similarly,

[ ][ ]

[ ] [ ]

Therefore, B is an orthogonal matrix

2. Show that the matrix P = [ ] is orthogonal [AU CBT D/J 2011]

49

Get useful study materials from www.rejinpaul.com


www.rejinpaul.com
MA2111/ Engineering Mathematics-I
Solution: To prove that:

[ ]; [ ]

[ ] [ ]= I

[ ][ ]

[ ] [ ]

Therefore, P is an orthogonal matrix

WORKING RULE FOR DIAGONALIZATION

A
[ORTHOGONAL TRANSFORMATION]:

Step 1: To find the characteristic equation


SH
YU
Step 2: To solve the characteristic equation
TH

Step 3:To find the eigen vectors


A

Step 4: If the eigen vectors are orthogonal, then form a normalized matrix N
PR

Step 5: Find

Step 6: Calculate AN

Step 7: Calculate D =

Problems:

1. Diagonalize the matrix [ ] [AU Jan 2013]

Solution: Let A = [ ]

50

Get useful study materials from www.rejinpaul.com


www.rejinpaul.com
MA2111/ Engineering Mathematics-I
The characteristic equation is where

Therefore, the characteristic equation is i.e.,

√ √

A
Hence, the eigen values of A are 0, 3, 15
SH
YU
To find the eigen vectors:
TH

[ ][ ] [ ]
A
PR

Case 1: When [ ][ ] [ ]

---------- (1)

-------- (2)

--------- (3)

Solving (1) and (2) by rule of cross-multiplication,

51

Get useful study materials from www.rejinpaul.com


www.rejinpaul.com
MA2111/ Engineering Mathematics-I
-6 2 8 -6

7 -4 -6 7

[ ]

Case 2: When [ ][ ] [ ]

-------- (1)

A
------- (2)

--------- (3)
SH
YU
Solving (1) and (2) by rule of cross-multiplication,
TH

-6 2 5 -6
A
PR

4 -4 -6 4

[ ]

Case 3: When [ ][ ] [ ]

---------- (1)

---------- (2)

52

Get useful study materials from www.rejinpaul.com


www.rejinpaul.com
MA2111/ Engineering Mathematics-I
---------- (3)

Solving (1) and (2) by rule of cross-multiplication,

-6 2 -7 -6

-8 -4 -6 -8

[ ]

A
[ ][ ]
SH
YU
[ ][ ] 4-2-2 = 0
TH

[ ][ ]
A
PR

Hence, the eigen vectors are orthogonal to each other

The Normalized matrix N = [ ]

[ ]

[ ]

[ ]

53

Get useful study materials from www.rejinpaul.com


www.rejinpaul.com
MA2111/ Engineering Mathematics-I
AN =

[ ] [ ] [ ][ ]

[ ] [ ] [ ]

[ ][ ] [ ] [ ]

[ ]

A
i.e., [ ]

The diagonal elements are the eigen values of A SH


YU
2. Diagonalize the matrix [ ] [AU Tvli. M/J 2011]
TH

Solution: Let A = [ ]
A
PR

The characteristic equation is where

Therefore, the characteristic equation is


2 1 -11 36 -36
0 2 -18 36
1 -9 18 0
54

Get useful study materials from www.rejinpaul.com


www.rejinpaul.com
MA2111/ Engineering Mathematics-I

√ √
=

Hence, the eigen values of A are 2, 3, 6

To find the eigen vectors:

[ ][ ] [ ]

Case 1: When [ ][ ] [ ]

A
---------- (1)
SH
YU
-------- (2)
TH

--------- (3)

Solving (1) and (2) by rule of cross-multiplication,


A
PR

-1 1 1 -1

3 -1 -1 3

[ ]

Case 2: When [ ][ ] [ ]

55

Get useful study materials from www.rejinpaul.com


www.rejinpaul.com
MA2111/ Engineering Mathematics-I
---------- (1)

-------- (2)

--------- (3)

Solving (1) and (2) by rule of cross-multiplication,

-1 1 0 -1

2 -1 -1 2

A
[ ]
SH
YU
Case 3: When [ ][ ] [ ]
TH

---------- (1)
A
PR

-------- (2)

--------- (3)

Solving (1) and (2) by rule of cross-multiplication,

-1 1 -3 -1

-1 -1 -1 -1

56

Get useful study materials from www.rejinpaul.com


www.rejinpaul.com
MA2111/ Engineering Mathematics-I

[ ]

[ ][ ]

[ ][ ] =0

[ ][ ]

Hence, the eigen vectors are orthogonal to each other

A
The Normalized matrix N =




;



SH


YU
[√ √ √ ] [√ √ √ ]
TH

√ √ √ √ √ √

AN = [ ] √ √ √ √ √ √
A

[√ √ √ ] [√ √ √ ]
PR

√ √ √ √ √ √ √ √

√ √ √ √ √ √ √ √
[ ]

[√ √ √ ] [√ √ √ ] [√ √ ]

i.e., [ ]

The diagonal elements are the Eigen values of A

QUADRATIC FORM- REDUCTION OF QUADRATIC FORM TO CANONICAL FORM BY


ORTHOGONAL TRANSFORMATION:

57

Get useful study materials from www.rejinpaul.com


www.rejinpaul.com
MA2111/ Engineering Mathematics-I
Quadratic form:

A homogeneous polynomial of second degree in any number of variables is called a quadratic form

Example: is a quadratic form in three variables

Note:

The matrix corresponding to the quadratic form is

[ ]

Problems:

A
1. Write the matrix of the quadratic form

Solution: Q =
SH
YU
[ ]
TH

Here ; ;
A

[ ]
PR

2. Write the matrix of the quadratic form [AU, Nov 2001]

Solution: Q = [ ]

[ ]

3. Write down the quadratic form corresponding to the following symmetric matrix [ ]

Solution: Let [ ] [ ]

58

Get useful study materials from www.rejinpaul.com


www.rejinpaul.com
MA2111/ Engineering Mathematics-I
The required quadratic form is

NATURE OF THE QUADRATIC FORM:

Rank of the quadratic form: The number of square terms in the canonical form is the rank (r) of the
quadratic form

Index of the quadratic form: The number of positive square terms in the canonical form is called the index
(s) of the quadratic form

Signature of the quadratic form: The difference between the number of positive and negative square terms

A
= s – (r-s) = 2s-r, is called the signature of the quadratic form

The quadratic form is said to be SH


YU
(1) Positive definite if all the eigen values are positive numbers
(2) Negative definite if all the eigen values are negative numbers
TH

(3) Positive Semi-definite if all the eigen values are greater than or equal to zero and at least one eigen
value is zero
A

(4) Negative Semi-definite if all the eigen values are less than or equal to zero and at least one eigen value
PR

is zero
(5) Indefinite if A has both positive and negative eigen values

Problems:

1. Determine the nature of the following quadratic form f( [AU April/May


2003]

Solution: The matrix of the quadratic form is Q = [ ]

The eigen values of the matrix are 1, 2, 0

Therefore, the quadratic form is Positive Semi-definite


59

Get useful study materials from www.rejinpaul.com


www.rejinpaul.com
MA2111/ Engineering Mathematics-I
2. Prove that the Q.F is indefinite [AU Tvli. M/J 2011]

Solution: The matrix of the quadratic form is Q = [ ]

| |

| |

Therefore, the quadratic form is indefinite

A
3. Discuss the nature of the quadratic form without reducing it to canonical
form
SH
YU
Solution: The matrix of the quadratic form is Q = [ ]
A TH

| |
PR

| |

Therefore, the quadratic form is positive definite

REDUCTION OF QUADRATIC FORM TO CANONICAL FORM THROUGH ORTHOGONAL


TRANSFORMATION [OR SUM OF SQUARES FORM OR PRINCIPAL AXES FORM]

Working rule:

Step 1: Write the matrix of the given quadratic form

Step 2: To find the characteristic equation


60

Get useful study materials from www.rejinpaul.com


www.rejinpaul.com
MA2111/ Engineering Mathematics-I
Step 3: To solve the characteristic equation

Step 4: To find the eigen vectors orthogonal to each other

Step 5: Form the Normalized matrix N

Step 6: Find

Step 7: Find AN

Step 8: Find D =

Step 9: The canonical form is [ ][ ] [ ]

A
Problems:

1. Reduce the quadratic form Q =


SH
orthogonal transformation [AU March 1996],[AU Trichy, Jan 2010]
into canonical form by an
YU
TH

Solution: The matrix of the Q.F is A =

[ ]
A
PR

i.e., A [ ]

The characteristic equation of A is where

Sum of the main diagonal elements = 6+3+3=12

The characteristic equation of A is

61

Get useful study materials from www.rejinpaul.com


www.rejinpaul.com
MA2111/ Engineering Mathematics-I
2 1 -12 36 -32
0 2 -20 32
1 -10 16 0
√ √

Therefore the eigen values are 2, 2, 8

To find the eigen vectors:

[ ][ ] [ ]

A
Case 1: When [ ][ ] [ ]
SH
YU
--------- (1)
TH

---------- (2)

---------- (3)
A
PR

Solving (1) and (2) by rule of cross-multiplication,

-2 2 -2 -2

-5 -1 -2 -5

[ ]

62

Get useful study materials from www.rejinpaul.com


www.rejinpaul.com
MA2111/ Engineering Mathematics-I
Case 2: When [ ][ ] [ ]

--------- (1)

---------- (2)

---------- (3)

All three equations are one and the same. Put . Then . Let = 1.

[ ]

A
Let [ ]. Since ,

SH
YU
[ ][ ] [ ][ ]
TH

------- (1)

-------- (2)
A
PR

Solving (1) and (2) by rule of cross-multiplication,

l m n

-1 1 2 -1

1 1 0 1

[ ]

63

Get useful study materials from www.rejinpaul.com


www.rejinpaul.com
MA2111/ Engineering Mathematics-I
√ √ √ √ √ √

The Normalized matrix N = √ √ √ √ √ √

[√ √ √ ] [√ √ √ ]

√ √ √ √ √

AN = [ ] √ √ √ √ √ √

[√ √ √ ] [√ √ √ ]

√ √ √ √ √
[ ]
√ √ √ √ √ √

A
[√ √ √ ] [√ √ √ ]

The canonical form is [ ][ ][ ] SH


YU
2. Reduce the quadratic form to a canonical form by an orthogonal reduction
TH

.Also find its nature. [AU Madurai, June 2011], [AU A/M 2011]
A
PR

Solution: The matrix of the Q.F is A =

[ ]

i.e., A [ ]

The characteristic equation of A is where

64

Get useful study materials from www.rejinpaul.com


www.rejinpaul.com
MA2111/ Engineering Mathematics-I
The characteristic equation of A is
1 1 0 -3 2
0 1 1 -2
1 1 -2 0
√ √

The eigen values are 1, 1, -2

To find the eigen vectors:

[ ][ ] [ ]

A
Case 1: When [ ][ ] [ ]
SH
YU

------- (1)
TH

------- (2)
A

------- (3)
PR

1 1 2 1

2 -1 1 2

[ ]

65

Get useful study materials from www.rejinpaul.com


www.rejinpaul.com
MA2111/ Engineering Mathematics-I
Case 2: When [ ][ ] [ ]

------- (1)

------ (2)

------ (3)

All three equations are one and the same.

Put , . Let . Then

[ ]

A
Let [ ]. Since is orthogonal to , SH
YU

[ ][ ] [ ][ ]
TH

-------- (1)
A
PR

--------- (2)

l m n

1 1 -1 1

-1 1 0 -1

[ ]

66

Get useful study materials from www.rejinpaul.com


www.rejinpaul.com
MA2111/ Engineering Mathematics-I
√ √ √ √ √ √

The Normalized matrix N = √ √ √


; √ √ √

[√ √ √ ] [√ √ √ ]

√ √ √ √ √ √

AN = [ ] √ √ √ √ √ √

[√ √ √ ] [√ √ √ ]

√ √ √ √ √ √ √ √
[ ]
√ √ √ √ √ √ √ √

A
[√ √ √ ] [√ √ √ ] [√ √ ]

i.e., [ ] SH
YU

The canonical form is [ ][ ][ ]


A TH

Nature: The eigen values are -2, 1, 1. Therefore, it is indefinite in nature.


PR

3. Reduce the given quadratic form Q to its canonical form using orthogonal transformation Q =
[AU Jan 2009]

Solution: The matrix of the Q.F is A =

[ ]

i.e., A = [ ]

The characteristic equation of A is where

67

Get useful study materials from www.rejinpaul.com


www.rejinpaul.com
MA2111/ Engineering Mathematics-I

The characteristic equation of A is


1 1 -7 14 -8
0 1 -6 8
1 -6 8 0
√ √

A
The eigen values are 1, 2, 4

To find the eigen vectors:


SH
YU
TH

[ ][ ] [ ]
A

Case 1: When [ ][ ] [ ]
PR

= 0 -------- (1)

-------- (2)

-------- (3)

Solving (2) and (3) by rule of cross multiplication, we get,

2 -1 0 2

-1 2 0 -1
68

Get useful study materials from www.rejinpaul.com


www.rejinpaul.com
MA2111/ Engineering Mathematics-I

[ ]

Case 2: When [ ][ ] [ ]

= 0 -------- (1)

-------- (2)

-------- (3)

A
SH
Solving (1) and (2) by rule of cross multiplication, we get,
YU
0 0 -1 0
TH

1 -1 0 1
A
PR

[ ]

Case 3: When [ ][ ] [ ]

= 0 -------- (1)

-------- (2)

-------- (3)

Solving (1) and (2) by rule of cross multiplication, we get,


69

Get useful study materials from www.rejinpaul.com


www.rejinpaul.com
MA2111/ Engineering Mathematics-I

0 0 -3 0

-1 -1 0 -1

[ ]

√ √ √

The Normalized matrix N = √ √ √


[ √ √ ]

A
[√ ] √ √
√ √

AN = [ ]
√ √ √

[ ]
SH
YU
√ √
√ √ √

[√ ] √ √
√ √
A TH

√ √ √ √ [ ]
PR

[ √ √ ][ √ √ ]

i.e., [ ]

The canonical form is [ ][ ][ ]

4. Reduce the quadratic form to the canonical form through an


orthogonal transformation and hence show that it is positive semi-definite. Also give a non-zero set of
values ( which makes the quadratic form zero. [AU M/J 2009], [AU Tvli. Jan 2009]

70

Get useful study materials from www.rejinpaul.com


www.rejinpaul.com
MA2111/ Engineering Mathematics-I

Solution: The matrix of the Q.F is A =

[ ]

i.e., A = [ ]

The characteristic equation of A is where

A
The characteristic equation of A is
SH
YU
√ √
TH

The eigen values are 0, 1, 3


A
PR

To find the eigen vectors:

[ ][ ] [ ]

Case 1: When [ ][ ] [ ]

------- (1)

------ (2)

71

Get useful study materials from www.rejinpaul.com


www.rejinpaul.com
MA2111/ Engineering Mathematics-I
------ (3)

Solving (1) and (2) by rule of cross multiplication, we get,

-1 0 1 -1

2 1 -1 2

[ ]

A
Case 2: When [ ][ ] [ ]
SH
YU
------- (1)
TH

------ (2)

------ (3)
A
PR

Solving (1) and (2) by rule of cross multiplication, we get,

-1 0 0 -1

1 1 -1 1

[ ]

72

Get useful study materials from www.rejinpaul.com


www.rejinpaul.com
MA2111/ Engineering Mathematics-I
Case 3: When [ ][ ] [ ]

--------- (1)

------------ (2)

---------- (3)

Solving (1) and (2) by rule of cross multiplication, we get,

-1 0 -2 -1

A
-1 1 -1 -1

SH
YU
[ ]
A TH

√ √ √ √ √ √

The Normalized matrix N =


PR

√ √ √ √

[√ √ √ ] [√ √ √ ]

√ √ √ √ √

AN = [ ] √ √
= √

[√ √ √ ] [ √ √ ]

√ √ √ √ √
[ ]
√ √ √

[√ √ √ ] [ √ √ ]

73

Get useful study materials from www.rejinpaul.com


www.rejinpaul.com
MA2111/ Engineering Mathematics-I
The canonical form is [ ][ ][ ]

Nature: The eigen values are 0, 1 and 3. Therefore, it is positive semi-definite.

To find the non-zero set of values which makes the quadratic form zero:

X = NY

√ √ √
[ ] [ ]
√ √

A
[√ √ √ ]

√ √ √
------- (1) SH
YU
------ (2)
√ √
TH

------- (3)
√ √ √
A

√ in (1), (2) and (3), we get,


PR

Put and

These values make the quadratic form zero.

Verification:

Q.F =

= = 1 + 2 +1 -2 -2 = 0

WORKING RULE FOR DIAGONALIZATION

[SIMILARITY TRANSFORMATION]:
74

Get useful study materials from www.rejinpaul.com


www.rejinpaul.com
MA2111/ Engineering Mathematics-I
Let A be any square matrix of order n.

Step 1: To find the characteristic equation

Step 2: To solve the characteristic equation

Step 3: To find the Eigen vectors

Step 4: Form the modal matrix P, its columns the Eigen vector of A

Step 5: Find

Step 6: Calculate AP

Step 7: Calculate D =

A
Problems based on similarity transformation:

1. Reduce the matrix [


SH
] to the diagonal form [AU April/May 2002]
YU
TH

Solution: Let A = [ ]. The given matrix is a non-symmetric matrix. Hence similarity


A

transformation is only possible.


PR

The characteristic equation of the given matrix A is where

| | | | | |

The characteristic equation of A is


1 1 -1 -5 5
0 1 0 -5
1 0 -5 0
75

Get useful study materials from www.rejinpaul.com


www.rejinpaul.com
MA2111/ Engineering Mathematics-I

i.e.,

Hence the Eigen values are 1, √ √

The given matrix is non-symmetric and the Eigen values are distinct. Hence we can find .

To find the Eigen vectors:

(A –

[ ][ ] [ ]

A
Case 1: When [ ][ ] [ ]
SH
YU

------------ (1)
TH

---------------- (2)
A

-------------- (3)
PR

Solving (1) and (2) by rule of cross multiplication, we get,

2 -2 -2 2

1 1 1 1

Hence [ ]

76

Get useful study materials from www.rejinpaul.com


www.rejinpaul.com
MA2111/ Engineering Mathematics-I

Case 2: When √ [ √ ][ ] [ ]

( √ ) ---------------- (4)

( √ ) --------------------- (5)

√ -------------------------- (6)

Solving (5) and (6), we get,

2-√ 1 1 2-√

A
-1 -√ -1 -1
SH
YU
√ √ √ √ √ √ ( √ ) √
TH

( √ ) √ ( √ ) √
A
PR

√ √

√ √


Hence [ ]


Case 3: When √ , we get, [ √ ][ ] [ ]

( √ ) ---------------- (7)

77

Get useful study materials from www.rejinpaul.com


www.rejinpaul.com
MA2111/ Engineering Mathematics-I
( √ ) --------------------- (8)

√ -------------------------- (9)

Solving (7) and (8), we get,

2 -2 -1+√ 2

2+√ 1 1 2+√

√ √ ( √ )( √ )

A
SH
√ √ √ √
YU
√ √ √ ( √ ) √ ( √ ) √ ( √ ) √
TH

√ √
A

√ √
PR


Hence [ ]

√ √
The modal matrix P = [ ]

To find

|P| = (√ ) ( √ ) √ √ √

Co-factor of P = [ √ √ √ ]

78

Get useful study materials from www.rejinpaul.com


www.rejinpaul.com
MA2111/ Engineering Mathematics-I

√ √
i.e., [ √ ]

√ √ √ √
AP = [ ][ ] [ √ √ ]=
√ √
√ √
[ √ √ ]
√ √

√ √ √ √

A
D= [ √ ][ √ √ ]



SH
√ √
YU
= [ √ √ √ √ √ √ ]

√ √ √ √ √ √
TH


= [ ] [ √ ]

A


PR

2. Diagonalize the matrix A = [ ]

Solution: The characteristic equation of A is where

| | | | | |

79

Get useful study materials from www.rejinpaul.com


www.rejinpaul.com
MA2111/ Engineering Mathematics-I
The characteristic equation of A is i.e.,
-1 1 -3 0 4
0 -1 4 -4
1 -4 4 0


i.e.,

Hence the Eigen values are -1, 2, 2.

The given matrix is a non-symmetric matrix and the Eigen values are not distinct. Hence
transformation is possible only if the Eigen vectors are distinct.

A
To find the Eigen vectors:

(A –
SH
YU
[ ][ ] [ ]
TH

Case 1: When [ ][ ] [ ]
A
PR

---------------- (1)

-------------- (2)

--------------- (3)

Solving (2) and (3) by rule of cross-multiplication, we get,

2 2 -1 2

-1 3 -1 -1

80

Get useful study materials from www.rejinpaul.com


www.rejinpaul.com
MA2111/ Engineering Mathematics-I

Hence [ ]

Case 2: When [ ][ ] [ ]

------------ (4)

--------------- (5)

-------------- (6)

A
SH
Solving (5) and (6) by rule of cross-multiplication, we get,
YU
-1 2 -1 -1
TH

-1 0 -1 -1
A
PR

Hence [ ]

We get only one Eigen vector corresponding to the repeated root . Hence similarity
transformation is not possible. Therefore, diagonalization is not possible.

81

Get useful study materials from www.rejinpaul.com


www.rejinpaul.com
MA2111/ Engineering Mathematics-I
UNIT 2 – SEQUENCES AND SERIES

PART - A

1. Define a sequence.

Solution: If in accordance with a definite rule, every number ‘ n ‘ of a series of natural numbers 1,2,3, . . . n
is put into correspondence with a certain real number , then the set of the ordered real quantities
, . . . is called a number sequence or simply a sequence.

The numbers will be referred mo as the terms of the sequence. The sequence may be
represented as { }={ } or

{ }={ }

Examples:

1. 1, 3, 5, 7, 9, . . . is a sequence of numbers.

A
2. -1, -2, -3, -4, . . . is a sequence of numbers

3. { } = SH
YU
4. { } = 2, 4, 6, 8, . . .
TH

2.Define limit of a sequence.

Solution: Let { } be the sequence of real numbers. Then approaches the limit L as n approaches
A

infinity, if for every > 0, there exists a positive integer N such that
PR

(n )

If approaches the limit L, then

3.Define convergence.

Solution: A sequence { } is said to be convergent if it has a finite limit.

i.e.,

Example: { } is a convergent sequence because

{ }

82

Get useful study materials from www.rejinpaul.com


www.rejinpaul.com
MA2111/ Engineering Mathematics-I
4.Define divergence.

Solution: If , then { } is divergent.

Example: { n } is divergent because .

5.When is a sequence said to be oscillatory? Give example.

Solution: If is not unique ( oscillates finitely ) or ( oscillates infinitely) then { } is


called an oscillatory sequence.

Examples:

1. { } oscillates finitely because


={
2. { } oscillates infinitely because

A
. =

6.Examine the convergence of the sequence {



} SH
YU
Solution: Here sn = Now

TH


A

( )
PR

( )

Hence the given sequence is convergent since the limit is 3.

7.Examine the convergence of the sequence { }

Solution: Here sn =

So ( )

83

Get useful study materials from www.rejinpaul.com


www.rejinpaul.com
MA2111/ Engineering Mathematics-I
This shows that sn does not tend to an unique limit. Hence the given sequence is oscillatory.

8.Define a bounded sequence.

Solution: A sequence { sn } is said to be bounded, if there exists a number k such that sn < k for every n.

9.Define a monotonic sequence.

Solution: A sequence { sn } is said to increase steadily or to decrease steadily according as sn+1 sn or sn+1
sn , for all values of n . Both increasing and decreasing sequences are called monotonic sequences.

A monotonic sequence always tends to a limit, finite or infinite. Thus, a sequence which is monotonic and
bounded is convergent.

10.Define a series.

Solution: Suppose we have an infinite sequence of numbers

A
The expression is called a series .

It is denoted by ∑
SH
YU
11.Examine the convergence of the series + +

Solution: Here = and = + + +


TH

=
A

=
PR

Hence the given series diverges to .

12.Examine the convergence of the series 1+ +

Solution: Here = 1+ +

84

Get useful study materials from www.rejinpaul.com


www.rejinpaul.com
MA2111/ Engineering Mathematics-I
= ( )= .

Since the partial sum sn converges to the given series converges to

13.Examine the convergence of the series

Solution: Here un = and this can be rewritten as

A
SH
YU
TH

=
A

=1–0=1
PR

=1-

Since the partial sum sn converges to the given series converges to

14.Examine the convergence of the series 2 – 3 + 1 + 2 -3 + 1+ 2 -3 + 1+ . . .

Solution : Construct the partial sum of the series as follows:

S1 = 2, s2 = 2 – 3= -1, s3 = 2 – 3 – 1 =0.

Hence we have, sn =2 -3 + 1 +2 -3 + 1 +2 -3 + 1+ . . . upto n terms.

= {

This implies that sn des not converge to a unique limit. Hence the series is oscillatory.
85

Get useful study materials from www.rejinpaul.com


www.rejinpaul.com
MA2111/ Engineering Mathematics-I
15. If then is the series of positive terms ∑ always convergent.

Solution: If then the series of positive terms ∑ need not necessarily be convergent. We
shall give a counter example to prove this.

Consider the series 1+


√ √ √

Sn = 1+
√ √ √
> 1+
√ √ √
=

Thus the series is divergent even though

16.State the Comparison test.

A
Solution: If two series of positive terms ∑ and ∑ be such that

(i)∑ converges SH
YU
( ii ) , for all n

Then the given series ∑ is convergent.


TH

17. State the Integral test.


A

Let ∑ be a series with positive and decreasing terms


PR

Let f be a non- negative decreasing function in [1,

f(1)=u1, f(2)=u2, f(3)=u3, . . . , f(n) = un

Then the improper integral ∫ and the series ∑ are both finite ( in this case ∑ is
convergent) or both infinite ( in this case ∑ is divergent).

18.Show that ∑ converges using the Integral test.

Solution: Let f(x)=

∑ =∑ ; f(x) > 0 and f(x) is decreasing in [1,

86

Get useful study materials from www.rejinpaul.com


www.rejinpaul.com
MA2111/ Engineering Mathematics-I

∫ ∫

=[ ] =[ ]

∫ converges to

By the integral test ∑ also converges.

19.Test the convergence of the Harmonic series 1+ using the Integral test.

Solution: The given series is ∑ 1+

A
∑ =∑ ; f(x) > 0 and f(x) is decreasing in [1,


SH

YU

=[ ] =[log log 1]
TH

=
A

∫ diverges in [1,
PR

Hence by the integral test ∑ also diverges.

20.State D’Alembert’s ratio test.

Solution: In a series with positive terms ∑

If ( i )

( ii )

( iii ) , and in this case we can use the comparison test.

21.Define alternating series.

Solution: A series in which the terms are alternatively positive or negative is termed as an alternating series.
87

Get useful study materials from www.rejinpaul.com


www.rejinpaul.com
MA2111/ Engineering Mathematics-I
Example: 2-

To test the convergence of the alternating series , we use a test known as the Leibnitz test.

22.Define Leibnitz test.

Solution: If in the alternating series

the terms are such that:

(i) and
(ii)

then the series is convergent.

If , then the series is oscillatory.

A
23.Discuss the convergence of the series 1- using Leibnitz test.
√ √ √

Solution: The terms are alternatively positive and negative


SH
YU
(i) 1>
√ √ √

(ii)

TH

Hence by Leibnitz test the series is convergent.

24.Test the convergence of the series ∑ using Leibnitz test.


A
PR

Solution: The given series ∑ = 1-

= 1-1+

=1–1+

The terms are alternatively positive and negative

(i) 1>
(ii)
Hence by Leibnitz test the series is convergent.

25.Discuss the convergence of the series 1- using Leibnitz test.

88

Get useful study materials from www.rejinpaul.com


www.rejinpaul.com
MA2111/ Engineering Mathematics-I
Solution: The given series 1- 1–

=1–

The terms are alternatively positive and negative

(i) 1>
(ii)
Hence by Leibnitz test the series is convergent.

26.Discuss the convergence of the series 2- using Leibnitz test.

Solution: The given series 2- with term is =

(i) 2>

A
(ii) . The second condition of Leibnitz test is not
satisfied, hence the series is not convergent (oscillatory).
SH
YU
27.Discuss the convergence of the series using Leibnitz test.

Solution: The given series =


TH

with term is =
A
PR

(i)
(ii)
Hence by Leibnitz test the series is convergent.

28.Discuss the convergence of the series 0< x < 1.

Solution: The given series is an alternating series with term is =

We Know that if 0 < x <1 , then >… and = 0.

This implies that (i)

(ii)
Hence by Leibnitz test the given series is convergent.
89

Get useful study materials from www.rejinpaul.com


www.rejinpaul.com
MA2111/ Engineering Mathematics-I
29.Test the convergence of the series ∑ .

Solution: The given series ∑ = 1-

The terms are alternatively positive and negative

(i) 1>
(ii) .

The second condition of Leibnitz rule is not satisfied, hence the series is not convergent (oscillatory).

30.When is a series said to be absolutely convergent?

Solution: If the series of arbitrary terms

be such that the series ∑ is convergent, then the series ∑

A
is
said to be absolutely convergent.

For example, consider the series 1- . SH


YU
It is absolutely convergent, since the series 1+ is known to be convergent.
TH

31.Define conditional convergence.

Solution: If ∑ is divergent but ∑ is convergent, then ∑ is said to be conditionally convergent.


A

For example, the series 1- is convergent, but the series of finite values 1+ is
PR

divergent, hence the original series is conditionally convergent.

PART - B

1.Prove that the series ∑ converges for k >1 and diverges for
k

Proof: The given series is ∑

Case 1: If k >1, then

90

Get useful study materials from www.rejinpaul.com


www.rejinpaul.com
MA2111/ Engineering Mathematics-I

( )

∑ = ( ) ( )+…

< 1+ ( )

Which is a geometric series with common ratio So, the series is convergent for k > 1.

Case 2: If k , then ∑

A
SH
YU
A TH
PR

∑ = 1+ ( ) ( )+…

>1+ which diverges to

Similarly, for k < 1 it diverges to Hence the given series converges for k >1 and diverges for k

2.Examine the convergence of the series

Solution: To find the term:

In denominator: 1, 2, 3, . . . = a+(n-1)d = 1+(n-1)1 = n which gives the first factor.

2, 3, 4, . . . = a+(n-1)d = 2+(n-1)1 = (n+1) which gives the second factor,

Where a denotes the first term and d denotes the common ratio.

91

Get useful study materials from www.rejinpaul.com


www.rejinpaul.com
MA2111/ Engineering Mathematics-I
Here =

Let us take

Then = = .

Since the series ∑ ∑ ∑ ∑ is


also convergent.

A
3.Examine the convergence of the series

Solution: To find the term:


SH
YU
In numerator: 1, 3, 5, . . . = a+(n-1)d = 1+(n-1)2 = (2n-1) which gives the first factor.

In denominator: 1, 2, 3, . . . = a+(n-1)d = 1+(n-1)1 = n which gives the first factor,


TH

2, 3, 4, . . . = a+(n-1)d = 2+(n-1)1 = (n+1) which gives the second factor,


A

3, 4, 5, . . . = a+(n-1)d = 3+(n-1)1 = (n+2) which gives the third factor,


PR

Where a denotes the first term and d denotes the common ratio.

Here = , and this can be written as

Let us take

92

Get useful study materials from www.rejinpaul.com


www.rejinpaul.com
MA2111/ Engineering Mathematics-I

Then =

( )
=
( )( )

Since the series ∑ ∑ ∑


∑ is also convergent.

4. Examine the convergence of the series

Solution: To find the term:

A
In numerator: 1, 4, 9, . . . = = which gives the first factor.

In denominator: 4, 7, 9, . . . SH
= a+(n-1)d = 4+(n-1)3 = (3n+1) which gives the first factor,
YU
7, 10, 13, . . . = a+(n-1)d = 7+(n-1)3 = (3n+4) which gives the second factor,

10, 13, 16, . . . = a+(n-1)d = 10+(n-1)3 = (3n+7) which gives the third factor,
TH

Where a denotes the first term and d denotes the common ratio.
A

Here = , and this can be written as


PR

=
( )( ) ( )( )( )

Let us take

93

Get useful study materials from www.rejinpaul.com


www.rejinpaul.com
MA2111/ Engineering Mathematics-I
( )( )( )
Then =

=
( )( )( )

Since the series ∑ ∑

∑ ∑ also divergent.


5.Test the convergence of the series ∑

√ √ √
Solution: The given series is

A

SH
= , and this can be written as

√ √
=
YU
( ) ( )

Let us take
A TH
PR

( )
Then =

=
( )

Since the series ∑ ∑ ∑ ∑ is


( )

also convergent.

94

Get useful study materials from www.rejinpaul.com


www.rejinpaul.com
MA2111/ Engineering Mathematics-I
√ √ √
6.Test the convergence of the series

√ √ √ √
Solution: The given series is

To find the term:

In numerator: √ ,√ ,√ ... =√

=√ =√ which gives the first factor.

In denominator: 3, 4, 5, . . . = a+(n-1)d = 3+(n-1)1 = (n+2) which gives the first factor,

where a denotes the first term and d denotes the common ratio.


= , and this can be written as

A
√ (√ ) (√ )

SH
√ √ √ √ √
=
(( ) ) (( ) )
YU
Let us take
A TH
PR

(√ )
√ √

(( ) )
Then =


√ √
=
( )

Since the series Since the series ∑ ∑ ∑



∑ is also convergent.

95

Get useful study materials from www.rejinpaul.com


www.rejinpaul.com
MA2111/ Engineering Mathematics-I
7.Test the convergence of the series

where a denotes the first term and d denotes the common ratio. Solution: The given series is

To find the term:

In numerator: 2, 3, 4, . . . = a+(n-1)d = 2+(n-1)1 = (n+1) which gives the first factor,

In denominator: 1, 2, 3, . . . = a+(n-1)d = 1+(n-1)1 = n which gives the first factor,

Here = , and this can be written as

Let us take

A
SH
YU
TH

Then =
A

= ( )
PR

Since the series ∑ ∑

∑ ∑ is convergent if p > 2 and divergent if p

8.Discuss the convergence of the series ∑

Solution: The given series is +

Case 1: For x < 1

96

Get useful study materials from www.rejinpaul.com


www.rejinpaul.com
MA2111/ Engineering Mathematics-I
=

Let us take

= .

We know that the series


∑ ∑ ∑ ∑ is also
convergent if x < 1.

Case 2: For x = 1, the series becomes which is a divergent series .

A
Case 3: For x > 1

=
SH
YU
Let us take
TH

= .
A
PR

We know that the series


∑ ∑ ∑ ∑ is
also convergent if x > 1.

Hence the given series converges for x < 1 and x > 1 and diverges for x = 1.

9.Test the convergence of the series ∑ √ –√ Test the convergence of the series

√ √

Solution: The given series can be written as

√ √
∑ √ –√ ∑ √ –√ ) (√ √ )

=∑ (√ √ )

97

Get useful study materials from www.rejinpaul.com


www.rejinpaul.com
MA2111/ Engineering Mathematics-I

√ √

= , and this can be written as


√ √

=
√ √
√ (√ )

Let us take

√ (√ )

A
Then =

=
(√ )
SH
YU
.
TH

Since the series


∑ ∑ ∑ ∑ √ –√ ) is also
A


divergent.
PR

10.Test the convergence of the series ∑ √

Solution: The given series is √ √ √ √

=√ , and this can be written as

– ( ) ( )
=√ √ ( ) √
( ) ( )

Let us take

98

Get useful study materials from www.rejinpaul.com


www.rejinpaul.com
MA2111/ Engineering Mathematics-I

( )

( )
( ) √
( )
Then =
( )

( )
= √
( )

√ .

Since the series ∑ ∑( ) is a geometric series with ratio

A
∑ ∑√ is also divergent.

11.Test the convergence of the series ∑ √


SH ( or )
YU
Test the convergence of the series∑

TH

Solution: Here un =√ and this can be written as


A

√ =( –
PR

=[ ( )] - n

= n[( ) ]

( )
=n[ ( ) ]

= n[ ]

= [ ]

= [ ]

99

Get useful study materials from www.rejinpaul.com


www.rejinpaul.com
MA2111/ Engineering Mathematics-I
Let us take

[ ]
Then =

= [ ]

Since the series


∑ ∑ ∑ ∑√ is also

A
convergent.

12.Test the convergence of the series ∑


SH
using the Integral test.
YU
Solution: Let f(x)= , for x
TH

∑ =∑ ; f(x) > 0 and f(x) is decreasing in [2,


A

∫ ∫
PR

Put log x = t

When x=2, t = log 2, when x= .

∫ ∫

=[ ] =[log log(log 2)]

∫ diverges in [2,

100

Get useful study materials from www.rejinpaul.com


www.rejinpaul.com
MA2111/ Engineering Mathematics-I
Hence by the integral test ∑ also diverges.

13.Discuss the convergence of the series ∑ using the Integral test.

Solution: Let f(x)=

∑ =∑ ; f(x) > 0 and f(x) is decreasing in [1,

∫ ∫

= ∫

= [ ] = [ ]

A
∫ ∫ = [ ]

= [ ]
SH
YU
=

∫ converges and by the integral test ∑ also converges.


TH

14.Examine the convergence of the series ∑ using the Integral test.


A

Solution: The given series is ∑


PR

1 sin 1+

∑ =∑ ; f(x) > 0 and f(x) is decreasing in [1,

∫ ∫

= ∫ ( )

=-( )

∫ ∫ = - ( )

= -( )
101

Get useful study materials from www.rejinpaul.com


www.rejinpaul.com
MA2111/ Engineering Mathematics-I
= -

∫ converges and by the integral test ∑ also converges.

15.Discuss the convergence of the series ∑ .

Solution: The given series is ∑ =

Here = and =

= =

A
= ( )

= ( ) SH
YU
( )

= <1 ( since e = 2.718…)


TH

Hence by ratio test the given series is convergent.


A

16.Discuss the convergence of the series ∑ .


PR

Solution: The given series is ∑ =

Here = and =

= =

= ( ) = .

= < 1.

102

Get useful study materials from www.rejinpaul.com


www.rejinpaul.com
MA2111/ Engineering Mathematics-I
Hence by ratio test the given series is convergent.

17.Test the convergence or divergence of the series ∑ ( ).

Solution: Consider the series ∑

Let = and =

= =

= ( ) = .

A
SH
= < 1.

Hence by ratio test the given series ∑ is convergent.


YU
Now, consider the series ∑ ∑ = 1+
TH

Clearly this series is convergent.

Since, ∑ is convergent and ∑ is convergent, by property of series ∑ is convergent.


A
PR

18.Examine the convergence of the series , x>0.

Solution: The term is =

and =

= =

=
( )

103

Get useful study materials from www.rejinpaul.com


www.rejinpaul.com
MA2111/ Engineering Mathematics-I
=

Hence by D’Alembert’s ratio test,

When x < 1 , the given series is convergent,

When x > 1 , the given series is divergent,

When x = 1,the ratio test fails.

Hence for x = 1 , we make use of the comparison test.

Put x=1 in un, then = and =

= = .

A
But since ∑ ∑ is convergent, by the comparison test ∑ is also convergent.

SH
Hence the given series converges for x ≤ 1 and diverges for x >
YU
19.Discuss the convergence of the series +…, x >0.
TH

Solution: The term is =

and =
A
PR

= =

= ×

Hence by D’Alembert’s ratio test,

When x < 1 , the given series is convergent

When x > 1 , the given series is divergent,

104

Get useful study materials from www.rejinpaul.com


www.rejinpaul.com
MA2111/ Engineering Mathematics-I
When x = 1,the ratio test fails.

Put x = 1 in un , then = .


Now =1

By the necessary condition for convergence the series diverges.

Hence the given series converges for x < 1 and diverges for x

20.Discuss the convergence of the series


√ √ √ √

Solution: The term is =


A
and =

= √
=


SH
YU


=

TH

( )
= √( .
( ) )
A
PR

= √ .

Hence by D’Alembert’s ratio test,

When < 1 , the given series is convergent,

When > 1 , the given series is divergent,

When = 1,the ratio test fails.

Hence for = 1 , we make use of the comparison test.

Put =1 in un, then = = and =


√ ( )

105

Get useful study materials from www.rejinpaul.com


www.rejinpaul.com
MA2111/ Engineering Mathematics-I
( )
= = .

But since ∑ ∑ is convergent, by the comparison test ∑ is also convergent.

Hence the given series converges for x ≤ 1 and diverges for x >

21.Discuss the convergence of the series

Solution: The term is =

and =

= =

A
=
SH
YU
=

=
TH

Hence by D’Alembert’s ratio test,


A

When x < 1 , the given series is convergent,


PR

When x > 1 , the given series is divergent,

When x = 1,the ratio test fails.

Now, we try to examine the series at x =1.

Put x = 1, the series becomes =( ) ( )

which is a divergent series .

Hence the given series converges for x < 1 and diverges for x

22.Check for convergence & absolute convergence of the series

Solution: The given series is


106

Get useful study materials from www.rejinpaul.com


www.rejinpaul.com
MA2111/ Engineering Mathematics-I
Next we construct the series of absolute terms as

| | | | | | | | | |

The series of absolute terms is

Which is the harmonic series with k=2, and hence the series is convergent. So the given series is absolutely
convergent.

23.Examine whether the series ∑ is absolutely convergent or conditionally convergent.

Solution: The given series ∑

A
Next we construct the series of absolute terms as

| | | | | | SH
YU
The series of absolute terms is
TH


A

Which is the harmonic series with k=1. This series is known to be divergent.
PR

Since the given series is an alternating series ∑ with

(i)
(ii)
By Leibnitz test the series is convergent.

Hence the given series is convergent, whereas the series of absolute terms is divergent. Therefore the series
is conditionally convergent.

24.Test for convergence & absolute convergence of the series ∑

Solution: The terms of the given series are alternately positive and negative with

107

Get useful study materials from www.rejinpaul.com


www.rejinpaul.com
MA2111/ Engineering Mathematics-I

The series is

Clearly,

(i)

(ii)

By Leibnitz test the series is convergent.

Next we construct the series of absolute terms as

A
| | | | | |

Let f(x)= , for x SH


YU
∑ =∑ ; f(x) > 0 and f(x) is decreasing in [2,
TH

∫ ∫
A
PR

Put log x = t

When x=2, t = log 2, when x= .

∫ ∫

= [ ] = (finite)

∫ converges in [2,

Hence by the integral test ∑ also converges, so the given series is absolutely convergent.

25.Test the series 1- for


√ √ √

108

Get useful study materials from www.rejinpaul.com


www.rejinpaul.com
MA2111/ Engineering Mathematics-I
(i) absolute convergence

(ii)conditional convergence

Solution: The terms are alternatively positive and negative

(i) 1>
√ √ √
(ii)

Hence by Leibnitz test the series is convergent.

Now, ∑ ∑ ∑

Is convergent since in k-series, for k = > 1 the series is convergent.


Hence the given series is absolutely convergent.

A
SH
YU
A TH
PR

109

Get useful study materials from www.rejinpaul.com


www.rejinpaul.com
MA2111/ Engineering Mathematics-I
Unit 3 Differential Calculus

Curvature The rate of bending of a curve in any interval is called the curvature of the curve in that interval

Curvature of a circle The curvature of a circle at any point on it equals the reciprocal of its radius.

Radius of curvature The radius of curvature of a curve at any point on it is defined as the reciprocal of the
curvature


{ ( ) }
Cartesian form of radius of curvature


Parametric equation of radius of curvature

A

SH
( )
Implicit form of radius of curvature =
YU
Centre of curvature

The circle which touches the curve at P and whose radius is equal to the radius of curvature and its centre is
TH

known as centre of curvature.


A

Equation of circle of curvature ̅ ̅


PR

Centre of curvature ̅ & ̅

Evolute The locus of the centre of curvature is called an evolute

Involute If a curve C1 is the evolute of C2 , then C2 is said to be an involute of a curve C1

Envelope A curve which touches each member of a family of curves is called envelope of that family
curves.

Envelope of a family of curves The locus of the ultimate points of intersection of consecutive members of
a family of curve is called the envelope of the family of curves.

Properties of envelope and evolute

110

Get useful study materials from www.rejinpaul.com


www.rejinpaul.com
MA2111/ Engineering Mathematics-I
Property:1 The normal at any point of a curve is a tangent to its evolute touching at the corresponding centre
of curvature.

Property:2 The difference between the radii of curvature at two points of a curve is equal to the length of the
arc of the evolute between the two corresponding points.

Property:3 There is one evolute ,but an infinite number of involutes

Property:4 The envelope of a family of curves touches at each of its point. The corresponding member of
that family.

Evolute as the envelope of normals The normals to a curve form a family of straight lines.we know that the
envelope of the family of these normals is the locus of the ultimate points of intersection of consecutive
normals.but the centre of curvature of a curve is also the point of consecutive normals.hence the envelope of

A
the normals and the locus of the centres of curvature are the same that is ,the evolute of a curve is the
envelope of the normals of the curve

Part - A
SH
YU
1. Find the radius of curvature of y= at x=0
TH


Ans:
A

y=
PR

y1= at x= 0 y1=1
y2= at x= 0 y2=1
⁄ ⁄
= =2√

2. Find the radius of curvature of at x = on the curve y = 4 sin x – sin 2x


Ans:

y1=4 cosx – 2 cos 2x at x= y1=2

y2= at x = y2=-4
⁄ ⁄

= =

111

Get useful study materials from www.rejinpaul.com


www.rejinpaul.com
MA2111/ Engineering Mathematics-I
3. Given the coordinates of the centre of curvature of the curve is given as ̅ 2a +3at2 ̅ -2at3
Determine the evolute of the curve
Ans: ̅ 2a +3at2 t2=( ̅ ⁄ )------------ 1
̅ -2at3 t3 = ̅ ------------ 2
( ̅ ⁄ )3 = ( ̅ 2

4( ̅ -2a)3=27a ̅ 2
The locus of the centre of curvature (evolute) is 4(x-2a)3=27a 2

4. Write the envelope of Am2+Bm+C=0, where m is the parameter and A,B and C are functions of
x and y.(NOV-08)

Solution: Given Am2+Bm+C=0……………………(1)

A
Differentiate (1) partially w.r.t. ‘m’

2Am+B=0 m=-B/2A………….(2)
SH
YU
Substitute (2) in (1) we get

A(-B/2A)2+B(-B/2A)+C=0
TH

AB2/4A2-B2/2A+C=0
A

AB2-2AB2+4A2C=0
PR

- AB2+4A2C=0

Therefore B2-4AC=0 which is the required envelope.

5. Find the radius of curvature at any point of the curve y=x2. (NOV-07)


( )
Solution: Radius of curvature

Given y=x2 y1= =2x

Y2 = =2

112

Get useful study materials from www.rejinpaul.com


www.rejinpaul.com
MA2111/ Engineering Mathematics-I

( )


( )
=

6. Find the envelope of the family of straight lines x sin + y cos = p, being the parameter. NOV-
07)

Solution: Given x sin + y cos = p……………. (1)

Differentiate (1) partially w.r.t. ‘ ’

X cos - y sin = o………………….(2)

A
Eliminate between (1) and (2)

X cos = y sin
SH
YU
TH

Tan =
A

Sin cos
√ √
PR

Substitute in (1)

x. + y. =p
√ √

√ =p

Squaring on both sides, x2 +y2=p2 which is the required envelope

7. What is the curvature of x2 +y2 - 4x-6y+10=0 at any point on it . (JAN-06)

Solution: Given x2 +y2 - 4x-6y+10=0

The given equation is of the form x2 +y2 +2gx+2fy+c =0

113

Get useful study materials from www.rejinpaul.com


www.rejinpaul.com
MA2111/ Engineering Mathematics-I
Here 2g =-4 g=-2

2f =-6 f=-3

Centre C(2,3), radius r =√

r=√

=√

Curvature of the circle =

There fore Curvature of x2 +y2 - 4x-6y+10=0 is


A
8. Find the envelope of the family of straight lines y= mx √ , where m is the parameter (JAN-06)

Solution: Given y= mx √
SH
YU
(y-mx)2=m2-1

Y2+m2x2 – 2mxy-m2+1=0
TH

m2 (x2-1)-2mxy+y2+1=0 which is quadratic in ‘m’


A

Here,A=x2-1 B=-2xy C=y2+1


PR

The condition is B2-4AC=0

4 x2y2-4(x2-1)(y2+1)=0

4 x2y2-4 x2y2-4x2+4y2+4=0

X2-y2=4 which is the required envelope

9. Find the curvature of the curve 2x2 +2y2 +5x-2y+1=0 (MAY-05,NOV-07)

Solution: Given 2x2 +2y2 +5x-2y+1=0

÷2 x2 +y2 +5/2x-y+1/2=0

114

Get useful study materials from www.rejinpaul.com


www.rejinpaul.com
MA2111/ Engineering Mathematics-I
Here 2g =5/2 g=5/4

2f=-1 f=-1/2 centre C (-5/4,1/2) radius r= √

=√


=√ =

Therefore Curvature of the circle 2x2 +2y2 +5x-2y+1=0 is


10. State any two properties of evolute . (MAY-05)

Solution: The normal at any point of a curve tangent to its evolute touching at the corresponding contre

A
of curvature. The difference between the radii of curvature at two points of a curve is equal to the length

SH
of the arc of the evolute between the two corresponding points.
YU
11. Define the curvature of a plane curve and what the curvature of a straight line. (JAN-05)

Solution: The rate at which the plane curve has turned at a point (rate of bending of a curve is called the
TH

curvature of a curve. The curvature of a straight line is zero.


A

12. Define evolute and involute .(JAN-05)


PR

Solution: The locus of centre of curvature of a curve (B1,B2,B3,…) is called evolute of the given curve.
If a curve C2 is the evolute of a curve C1 ,then C1 is said to be an involute of a curve C2.

13. Find the radius of curvature of the curve x2 +y2 -6x+4y+6=0 (NOV-08)

Solution: Given X2 +y2 - 6x+4y+6=0

The given equation is of the form x2 +y2 +2gx+2fy+c =0

Here 2g =-6 g=-3

2f =4 f=2

Centre C(3,-2), radius r =√

115

Get useful study materials from www.rejinpaul.com


www.rejinpaul.com
MA2111/ Engineering Mathematics-I
r=√

=√

Radius of Curvature of the circle = radius of the circle=√

14. Find the envelope of the family of circles (x- )2+y2=4 ,where is the parameter.(MAY-07)

Solution: Given (x- )2+y2=4

X2-2 x+ 2-4 +y2=0

2
-2 (x+2)+x2+y2=0 which is quadratic in

The condition is B2-4AC=0

A
Here A=1 B=-2 (x+2) C= x2+y2

4(x+2)2-4(x2+y2)=0
SH
YU
x2-4x+4- x2-y2=0
TH

y2+4x=4 which is the required envelope.


A

15. Define evolute .(MAY-07).


PR

Solution: The locus of centre of curvature ( ̅ , ̅) is called an evolute .

16. Find the envelope of the family of straight lines y=mx+ for different values of ‘m’. (NOV-07)

Solution: Given y=mx+

m2x-my+a=0 which is quadratic in ‘m’

The condition is B2-4AC=0

Here A=x B=-y C=a

Y2-4ax=0

116

Get useful study materials from www.rejinpaul.com


www.rejinpaul.com
MA2111/ Engineering Mathematics-I
There fore y2=4ax which is the required envelope.

17. Find the envelope of the line +yt=2c,where ‘t’ is the parameter.(NOV-02,05).

Solution: Given +yt=2c

Yt2-2ct+x=0 which is quadratic in ‘t’

The condition is B2-4AC=0

Here A=y B=-2c C=x

C2-xy=0

There fore xy=c2 which is the required envelope.

A
SH
18. Find the radius of curvature of the curve y=c cosh(x/c)at the point where it crosses the y-axis.
(NOV-05)
YU

( )
Solution: Radius of curvature
TH

Given y=c cosh(x/c) and the curve crosses the y-axis. (i.e.)x=0 implies y=c.
A

Therefore the point of intersection is (0,c)


PR

=c cos h(x/c)(1/c)=cos h (x/c)

(0,c)=cos 0=1

=cos h(x/c)(1/c)

(0,c)=1/c


=c2√

19. Find the radius of curvature of the curve xy=c2at (c,c).(NOV-02)

117

Get useful study materials from www.rejinpaul.com


www.rejinpaul.com
MA2111/ Engineering Mathematics-I

( )
Solution: Radius of curvature

Given xy=c2

=x +y=0

=- implies (c,c)=-1

=-[ ]

(c,c)= =-[ ]

A



SH
YU
TH

PART-B
A

⁄ ⁄ ⁄
1. Find the radius of curvature at the point on the curve .
PR

(NOV-07,MAY-08, JUNE 2009)

Solution: Given x= ……………………(1)

Y= …………………..(2)

Differentiate (1) and (2) w.r.t

118

Get useful study materials from www.rejinpaul.com


www.rejinpaul.com
MA2111/ Engineering Mathematics-I

= ( )

=-sec2


{ ( ) }
Radius of curvature

A
SH

( ) ⁄
=
YU
=3a sin
TH

2. Find the radius of curvature of the curve at the point (-a,0).(NOV-08).


A
PR


{ ( ) }
Solution: Radius of curvature

Given

Differentiate w.r.t. ‘x’


( ) ( )
2y

119

Get useful study materials from www.rejinpaul.com


www.rejinpaul.com
MA2111/ Engineering Mathematics-I

{ ( ) }

[ ( ) ] [ ]


{ }

A
6. Find the radius of curvature at the point (a,0)on the curve
NOV 2010) SH .(MAY-07, JAN 2010,
YU

{ ( ) }
Solution: Radius of curvature
TH

Given
A
PR

Differentiate w.r.t.’x’

2xy +y2.1=-3x2 ………………(1)


{ ( ) }
Therefore

120

Get useful study materials from www.rejinpaul.com


www.rejinpaul.com
MA2111/ Engineering Mathematics-I
Differentiate (2) w.r.t.’y’.

[( )( ) ( )]

[( ) ]


{ ( ) } { } ⁄
Therefore radius of curvature ⁄

(since the radius of curvature is non-negative)

7. Find the curvature of the parabola y2=4x at the vertex.(NOV-07)

A

{ ( ) }
Solution: Radius of curvature

Given; y2=4x
SH
YU
Differentiate w.r.t.’x’
TH

2y =4
A
PR

=2/y


{ ( ) }
Therefore

Differentiate (1) w.r.t.’y’.


121

Get useful study materials from www.rejinpaul.com


www.rejinpaul.com
MA2111/ Engineering Mathematics-I

{ } ⁄
Therefore =2

Curvature K=1/ =1/2

8. Find the radius of curvature of the curve 27ay2= 4x3 at the point where the tangent of the curve
makes an angle 450 with the X- axis.

Solution; Let (x1,y1) be the point on the curve at which the tangent makes an angle 450 with the X- axis.

(x1,y1) =Tan 45o=1-------------------------------- (1)

A
Given 27ay2= 4x3

Differentiate w.r.t.’x’ SH
YU
54ay =12x2 =
TH

(x1,y1) = = -----------------------------------(2)
A
PR

(x1,y1) =Tan 45o=1=

Gives -----------------------------(3)

As ( x1,y1) lies on the curve 27ay21= 4x31 ---------------------------------(4)

Using gives x1= 3a

And using (3) gives y1= 2a

Y1 at (3a,2a)= 1

Y2= [ ]

122

Get useful study materials from www.rejinpaul.com


www.rejinpaul.com
MA2111/ Engineering Mathematics-I
Y2= [ ]=1/6a

⁄ ⁄
Therefore radius of curvature ⁄

7. Find the evolute of the rectangular hyperbola xy=c2.(JAN-06,NOV-08)

Solution: The equation of the given curve is xy=c2……….(1)

The parametic form of (1) is

X=ct; y=

A
=c; =c ( )=-

Y1=
SH
YU

Y2= ( ) ( )
TH

= ( )
A
PR

The co-ordinates of the center of curvature Is ̅ ̅

Where ̅

( ⁄ ) ( ) ( )
( )
( ⁄ )

̅ ( )………………………..(2)

123

Get useful study materials from www.rejinpaul.com


www.rejinpaul.com
MA2111/ Engineering Mathematics-I
( ) ( )
= ( )
( ⁄ )

̅ ( )………………………………(3)

Eliminating ‘t’ between (2)and(3),

(2)+(3)gives

̅ ̅ ( ) ( ) ( )

̅ ̅ ( ) ……………………………(4)

(2)-(3)gives

A
̅ ̅ ( ) (
SH
) ( )
YU
̅ ̅ ( ) …………………..(5)
TH

(4)2/3-(5)2/3gives
A


⁄ ⁄
̅ ̅ ̅ ̅ ( ) [( ) ( ) ]
PR


= ⁄

⁄ ⁄ ⁄
Therefore ̅ ̅ ̅ ̅

The locus of centre of curvature ̅ ̅ is

⁄ ⁄ ⁄
which is the required evolute of the rectangular hyperbola xy=c2.

9. Find the radius of curvature for the curve r=a(1+cos )at is a constant.

(NOV-07,08)

124

Get useful study materials from www.rejinpaul.com


www.rejinpaul.com
MA2111/ Engineering Mathematics-I
Solution: Given r=a(1+cos )

R’=-a sin

R’’ =cos

The radius of curvature in polarform is


( )
=


{ }
=[ ]

⁄ ⁄

=

A
=
√ ⁄

SH
YU

at is
TH

Also,
A

Therefore, = .
PR

8. Considering the evolute as the envelope of normal’s, find the evolute of the parabola x2=4ay.(NOV-
08)

Solution: Given x2=4ay

The parametric equations are x=2at, y=at2

=2a =2at

125

Get useful study materials from www.rejinpaul.com


www.rejinpaul.com
MA2111/ Engineering Mathematics-I
M=

We know that the equation of normal to the curve is,

y-y1= (x-x1)

y-at2= (x-2at) yt-at3=-x+2at

x+yt=at3+2at……………….(1)

Differentiate (1) partially w.r.t.’t’we get


Y=3at2+2a t2 ( )

A
Substitute the value of ‘t’ in (1)

y( )

+x=a( )

+2a( )
⁄ SH
YU
⁄ ⁄
x =( ) [ ( ) ] ( ) [( ) ]
TH


=( ) [ ]
A
PR


√ √

Squaring on both sides, we get

which is the required evolute.

9. Obtain the evolute of the parabola y2=4ax. (NOV-07, JAN 10,JUNE 2010, JAN 2011,JAN 2012, JUNE
2012)

Solution: Given y2=4ax………………………(1)ss

The parametric equations are x= at2, y=2at

=2at =2a
126

Get useful study materials from www.rejinpaul.com


www.rejinpaul.com
MA2111/ Engineering Mathematics-I
=y1

Y2= ( ) ( )

= ( ) =

The co-ordinates of the center of curvature Is ̅ ̅

Where ̅

( ⁄) ( )
( )
( ⁄ )

̅ ……………………..(2)

A
̅
SH
YU
̅ ( )
( ⁄ )
TH

̅ ………………………………(3)
A

Eliminating ‘t’ between (2)and(3),


PR

̅
(2) gives

̅
(3) gives

̅ ̅
( ) ( )

̅ ̅

The locus of centre of curvature ̅ ̅ is

which is the required evolute.

127

Get useful study materials from www.rejinpaul.com


www.rejinpaul.com
MA2111/ Engineering Mathematics-I
10. Find the equation of the envelope of .(NOV-02,07, NOV 2010, JAN

2013)

Solution: Given that …………………(1)

And ……………..(2)

Differentiate (1)and(2) w.r.t ‘b’

………………..(3)

2a +2b=0…………………………(4)

……………..(5)

A
(3)gives

(4)gives ……………………(6)
SH
YU
From (5)and (6)
TH

⁄ ⁄
A
PR

⁄ ⁄

⁄ ⁄
Substitute in (2) we get,

⁄ ⁄ ⁄
Therefore which is the required envelope.

11. Find the equation of circle of curvature of the parabola y2=12x at the point (3,6).(NOV-07,08)

Solution: The equation of circle of curvature is ̅ ̅

Where, ̅

128

Get useful study materials from www.rejinpaul.com


www.rejinpaul.com
MA2111/ Engineering Mathematics-I
̅

Given y2=12x

Differentiate w.r.t.’x’ we get

2y =12 implies

Y1= (3,6)=1

A
Y2= (3,6)=-1/6


SH ⁄


YU

√ ( can not be negative)


TH

̅
A
PR

=3 ⁄

̅

Therefore,the equation of circle of curvature is

12. Find the radius of curvature at ‘t’ on x=etcost,y=etsint.(JAN-06)


Solution: Radius of curvature

Given

129

Get useful study materials from www.rejinpaul.com


www.rejinpaul.com
MA2111/ Engineering Mathematics-I
X’=

Y’=

X’’=

Y’’=


The radius of curvature is


[ ] [ ]

A

[ ] ⁄ ( )

SH
= [ ]

√ .
YU

13. Find the evolute of the ellipse .(MAY-05,07)


TH

Solution: The given curve is


A
PR

The parametric equations are x=acos ,y=bsin

Y1=

Y2= ( ) ( ) ( )

Y2

The Co-ordinate of centre of curvature is ̅ ̅

130

Get useful study materials from www.rejinpaul.com


www.rejinpaul.com
MA2111/ Engineering Mathematics-I
Where ̅

( )( )
= acos - ( )

= acos

= ( ) ( )

̅ ( ) ………………….(1)

A
SH
̅
YU
( )
= + ( )
TH

[ ]
A

[ ] [ ]
PR

̅ [ ]………………………(2)

Eliminating ‘ ’ between (1) and (2),we get

⁄ ⁄
̅ ̅
( ) ( )

we know that,

⁄ ⁄ ⁄ ⁄
̅ ̅ ̅ ̅
( ) ( )
⁄ ⁄

131

Get useful study materials from www.rejinpaul.com


www.rejinpaul.com
MA2111/ Engineering Mathematics-I
⁄ ⁄ ⁄
̅ ̅

⁄ ⁄ ⁄
The locus of ̅ ̅ is which is the evolute of the ellipse

14. Find the envelope of where l and m are connected by and a,b are constants.

(MAY – 05, NOV-05,JAN 2012)

Solution: Given that ………………..(1)

…………………(2)

Differentiating (1) w.r.t.’m’

A
( ) ( )

SH
…….(3)
YU

Differentiating (2) w.r.t.’m’


A TH
PR

…………..(4)

From (3) and (4)

√ √ substitute in equation (2) ,

132

Get useful study materials from www.rejinpaul.com


www.rejinpaul.com
MA2111/ Engineering Mathematics-I

√ √

√ √ which is the required envelope.

15. Find the points on the parabola at which the radius of curvature is 4 √ . (MAY – 05)

Solution: Given …………….(1)

Let, P (a,b) be the point on the curve at where √

A
Differentiate (1) w.r.t. ‘x’

Y1=2y SH
YU
TH

Y2=
A
PR

⁄ ⁄


But, √


8 √

a+1=2 a=1,

The points are (1,2),(1,-2)

133

Get useful study materials from www.rejinpaul.com


www.rejinpaul.com
MA2111/ Engineering Mathematics-I
16. Considering the evolute of a curve as the envelope of its normals find the evolute of .

(NOV-02,05,MAY-05)

Solution: The given curve is

The parametric equations are x=acos ,y=bsin

m=

We know that the equation of the normal is y-y1= (x-x1)

A
SH
YU
A TH

,we get
PR

…………..(1)

Differentiate (1) partially w.r.t.’ ’, we get

134

Get useful study materials from www.rejinpaul.com


www.rejinpaul.com
MA2111/ Engineering Mathematics-I
⁄ ⁄

√ ⁄ ⁄ √ ⁄ ⁄

Substitute in equation (1),we get

√ ⁄ ⁄ √ ⁄ ⁄
⁄ ⁄

√ ⁄ ⁄ ⁄ ⁄


⁄ ⁄
[ ]

⁄ ⁄ ⁄
which is the required evolute of the ellipse.

A
SH
17. Find the circle of curvature at (3,4)on xy=12.(JAN-05, JAN 2010)

Solution: The equation of circle of curvature is ̅ ̅


YU
Where, ̅
TH

̅
A
PR

Given xy=12

Differentiate w.r.t.’x’ we get

x implies

( )
Y1= (3,4)=-4/3

( )
Y2= (3,6)

135

Get useful study materials from www.rejinpaul.com


www.rejinpaul.com
MA2111/ Engineering Mathematics-I


⁄ ( )


=3 ( )

̅ ( )

A
Therefore, the equation of circle of curvature is ( ) ( ) ( ) .

SH ⁄ ⁄ ⁄
YU
18. Find the evolute of the four cusped hyper cycloid .(JAN-05, NOV-07)

⁄ ⁄ ⁄
Solution: The equation of the given curve is ………………..(1)
TH

The parametric equations are x = a cos 3 , y = a sin 3


A
PR

Y1= / = -tan

Y2= - sec2 / =( (sec4 θ cosecθ))/3a

̅ = a cos 3 - ( 1 + tan 2

= a cos 3 + ------------------------------------------(2)

̅ a sin 3 ( 1 + tan 2

136

Get useful study materials from www.rejinpaul.com


www.rejinpaul.com
MA2111/ Engineering Mathematics-I
3
a sin + -----------------------------------------(3)

Eliminate from 2 & 3

̅ + ̅ = a cos 3 + + a sin 3 +

3
= a( cos + sin ------------------------------------(4)

̅ - ̅ = a cos 3 + - a sin 3 +

3
= a( cos - sin ------------------------------------(5)

⁄ ⁄ ⁄ ⁄ ⁄
̅ ̅ + ̅ ̅ = (( ) +( )


= (2)

A
The locus of centre of curvature is ⁄
+
SH⁄
= ⁄
(2)
YU
19. Find the radius of curvature at the origin of the cycloid x = a ( + sin ) and y = a( 1- cos ).
(MAY’07, Nov ’08, JUNE 10, JUNE’12,JAN 13)
TH

Given: x = a ( + sin ) , y = a( 1- cos ).


A

x’ =a( 1 + cos ) y’ = a( sin )


PR

x” = -a sin  y” = a cos 


The radius of curvature is

  ⁄
= = 4a cos
   

At = 0

20. Find the envelope of the straight lines represented by the equation x cos α + y sin α = a sec α , α is
the parameter (Nov’ 07)

Solution: Given x cos α + y sin α = a sec α

137

Get useful study materials from www.rejinpaul.com


www.rejinpaul.com
MA2111/ Engineering Mathematics-I
Divided by cos α

x + y tan α = a sec2α

x + y tan α = a ( 1 + tan2α)

a tan2α – y tan α + a –x = 0

which is quadratic in tan α

A = a, B = -y C = a-x

The envelope is given by B2 – 4AC = 0

y2 = 4a(a-x) which is the required envelope

A
21. Prove that the evolute of the curve x = a (cos  + log tan ), y=a sin  is the catenary y = a cosh

Nov ‘05 SH
YU
Solution : x = a (cos  + log tan )
TH

x’ = a(-sin  + )
A

= a cot  cos 
PR

y=a sin 

y’ = a cos 

y1 = = tan 

y2 = =¼ (sec 4  sin )

138

Get useful study materials from www.rejinpaul.com


www.rejinpaul.com
MA2111/ Engineering Mathematics-I
= a (cos  + log tan )- ( 1 + tan 2

̅ =a log tan ………………..(1)

= a sin + ( 1 + tan 2

̅ ……………….(2)

Eliminate ‘’from (1) and (2)

A
̅⁄
tan = ………………….(3)

̅ (
=
̅⁄ ̅̅̅̅̅⁄
) ̅ SH
YU
y = a cosh which is the required evolute
A TH
PR

139

Get useful study materials from www.rejinpaul.com


www.rejinpaul.com
MA2111/ Engineering Mathematics-I
Unit 4 Functions of several Variables

I. Problems based on Partial Derivatives


II. Problems based on Euler`s Theorem
III. Problems based on Total Derivatives-Differentiation of Implicit Function
IV. Problems based on Jacobian
V. Problems based on Taylor`s and Laurent Series
VI. Problems based on Maxima and Minima for Functions of Two Variables
VII. Problems based on Lagrangian Multiplier

Partial Derivatives

Partial Derivatives:

A
Let be a function of two Variables x and y, If we keep y as a constant and Vary x alone , then z
is a function of x only ,
SH
YU
The derivative of z w.r.to x, treating y as a constant is called the partial derivatives w.r.to x and it is denoted
by the symbols
A TH
PR

Notation:

Successive Partial Differentitation:

Let , then being the function of x and y can further be differentiation partially with

respect to x and y.

( ) ( )

( ) ( )

Problems:

140

Get useful study materials from www.rejinpaul.com


www.rejinpaul.com
MA2111/ Engineering Mathematics-I
1. If u = find

Solution:

2. Find if where and

Solution:

3. If show that

Solution: Given

A
SH
YU
TH

Adding (1), (2),and (3) we get


A
PR

4. show that

Solution: Let ,

(1)

(2)

(3)

(1)+ (2)+ (3)


141

Get useful study materials from www.rejinpaul.com


www.rejinpaul.com
MA2111/ Engineering Mathematics-I
5. If ( ), Show that

Solution: Given

z is a homogeneous function of degree n=2

As z is homogeneous function of order n=2, it satisfies the Eulers equation

A
SH
YU
A TH

6. If u = , find
PR

Sol: given u =

( ) ( )

( )

Euler`s Theorem for Homogeneous Function

Euler`s Theorem: If u be a homogeneous function of degree n an x and y then ( ) ( )

142

Get useful study materials from www.rejinpaul.com


www.rejinpaul.com
MA2111/ Engineering Mathematics-I
Problems:

1. Verify Eulers theorem for the function

Solution: Given

This is a homogeneous function of degree 2.

A
Adding (1) and (2) we get
SH
YU
Hence Eulers theorem is verified.
TH

2. If √ , Show that


A

Solution: Given
PR

As z is a homogeneous function of order n = 1, it satisfies the Eulers theorem

(1)

(1) (2)

By Eulers theorem of second order

143

Get useful study materials from www.rejinpaul.com


www.rejinpaul.com
MA2111/ Engineering Mathematics-I
(3)

, put in (3)

√ √
3. If u = , prove that
√ √


√ √ √ ( )

Sol: given √
= ( )
√ √ √ ( )

is a homogeneous function of degree 0.

Hence by Euler’s theorem, we have xu=0

A
SH
Total Derivatives-Differentiation of Implicit Function
YU
Total Derivative: If , then we can express u as a function of

t alone by substituting the value of x and y in Thus we can find the ordinary derivatives which
TH

is called the total derivative of u distinguish it from partial derivatives


A

Now to find the without actually substituting the values of x and y in we establish the
PR

following formula

Problems:

1. Find if where and

Solution:

2. If where and find and

Solution:

144

Get useful study materials from www.rejinpaul.com


www.rejinpaul.com
MA2111/ Engineering Mathematics-I

3. Find if

Solution: Let

( ) ( )

A
4. Find ( )
SH
YU
Ans :
TH

( ) ( )( ) ( )( )
A

5. If z be a function of x and y and u and v are other two variables, such that
PR

show that ( )

Solution: z may be represented as the function of u,v

( )( ) (1)

Similarly

145

Get useful study materials from www.rejinpaul.com


www.rejinpaul.com
MA2111/ Engineering Mathematics-I
(2)

(1)+(2) ( )

6. ( )

Ans: Here Z is a composite function of u and v

……………………………………(1)

………………………………………..(2)

A
Now

Sub these values in (1) & (2) We get


SH
YU
………………………………..(3)
TH

Now ( )
A

Which implies ( )……………..(4)


PR

(3)x(4) We get

( ) ( )( )

……………………………..(A)

Similarly we get ………………………(B)

(A)+(B) Gives

146

Get useful study materials from www.rejinpaul.com


www.rejinpaul.com
MA2111/ Engineering Mathematics-I

( )

14. If u is a function x and y and x and y are functions of r and  given by


(a) x  e r cos  , y  e r sin  shown that
 2u  2u 2 r   u  2u 
2
  e  2  
x 2 y 2  r  2 
(b) x  r cos  , y  r sin  prove that
2
 z   z   z  1  z 
2 2 2

       2  
 x   y   r  r  r 
  2 z    2 z    2 z  1   2 z  1  z 
and  2    2    2   2  2     .
 x   y   r  r    r[   
Solution

A
(a) Here u is a function of x, y which is given as a function of r and  .

SH
x r x
 e cos   x;  e r sin    y;
r 
y y
 e r sin   y;  e r cos   x
YU
r 
x x y y
 x;   y;  x and y
r   r
TH

u u x u y u u
Now  .  . x y ...(1)
r x r y r x y
A

u u x u y u u
PR

 .  .  y  x ...(2)
 x  y  x y
From (1), we get ,
  
x y ...(3)
r x y

147

Get useful study materials from www.rejinpaul.com


www.rejinpaul.com
MA2111/ Engineering Mathematics-I
From (2) , we get ,
  
 y  x ...(4)
 x y
 2u   u      u u 
Now,   x  y  x  y 
r 2
r  r   x y  x y 
 2u 2  u
2
 2u 2  u
2
x  2 xy y ...(5)
r 2 x 2 xy y 2
 2u   u      u u 
    y  x   y  x 
 2
     x y  x y 
 2u 2  u
2
 2u 2  u
2
 y  2 xy  x ...(6)
 2 x 2 xy y 2
Adding (5) and (6) we get ,
 2u  2u 2  u
2
2  u
2
  (x  y ) 2  (x  y ) 2
2 2

r 2  2 x y

A
  2u  2u    2u  2u 
 ( x 2  y 2 )  2  2   e2 r  2  2 

i.e., 
x 2 y 2
e  2  2
 r
 x
 2u  2u 2 r   2u  2u 
 
y   x y 
SH
YU
x x y y
(b)  cos  ;  r sin  ;  sin  ;  r cos 
r  r 
TH

z z x z y z z
 .   cos  .  sin  .
r x r y r x y
z z x z y z z
A

   r sin   r cos 
 x  y  x y
PR

2 2
 z  1  z   z z   z z 
2

   2    cos   sin     sin   cos  


 r  r     x y   x y 
2
 z   z 
2

   
 x   y 

148

Get useful study materials from www.rejinpaul.com


www.rejinpaul.com
MA2111/ Engineering Mathematics-I
 z   z 
2
  
r 2 r  r 
  z z 
 cos   sin  
r  x y 
  2 z x  2 z y   z x  2 z y 
 cos   2    sin   xy r  y 2 r 
 x r yx r   
 z
2
 z
2
 z
2
 cos 2  2  sin 2  2  2sin  cos 
x y xy
2 z   z 
  
 2
   
  z z 
   r sin   r cos  
  x y 
z   2 z x  2 z y 
  r cos   r sin   2  
x  x  yx  

A
 r sin 
z
y
  2 z  2 z y 
 r cos    2
  y  
 SH
YU
2 z 2 2 z 2 z  z z 
 r 2 sin 2   r cos 2
  2 r 2
sin  cos   r  cos   sin  
x 2
y 2
xy  x y 
TH

2 z 1 2 z 2 z 2 z 1  z z 
 2  2  2   cos   sin  
r 2
r  2
x y r  x y 
 2 z  2 z 1 z
A

  
x 2 y 2 r r
PR

 2 z  2 z  2 z 1  2 z 1 z
   
x 2 y 2 r 2 r 2 r 2 r r

Jacobian

Defn : are functions of n variables the the Jacobian of the transformation


from is defined by

149

Get useful study materials from www.rejinpaul.com


www.rejinpaul.com
MA2111/ Engineering Mathematics-I

| |

| |

| |

and is denoted by the symbol or

Problems:

1. Prove that the functions are are functionally dependent.

A
SH
Solution:
YU
|| || || ||
TH

u and v are not independent, u ,v are functionally dependent.


A
PR

2. Find the Jacobian of u = xyz, v = xy + yz + zx, w = x + y + z

Solution: .

3. If find

Solution:

150

Get useful study materials from www.rejinpaul.com


www.rejinpaul.com
MA2111/ Engineering Mathematics-I
  y1 , y2 , y3 
15. Find the value of , if y1  1  x1  , y2  x1 1  x2  , y3  x1 x2 1  x3  .
  x1 , x2 , x3 
Solution :
y1 y1 y1
x1 x2 x3
  y1 , y2 , y3  y2 y2 y2

  x1 , x2 , x3  x1 x2 x3
y3 y3 y3
x1 x2 x3
1 0 0
 1  x2  x1 0
x2 1  x3  x1 1  x3   x1 x2
  1  x1   x1 x2 
  x12 x2

A
Hence the solution.

SH
YU
A TH
PR

151

Get useful study materials from www.rejinpaul.com


www.rejinpaul.com
MA2111/ Engineering Mathematics-I
  x, y , z 
16. If u  xyz , v  x 2  y 2  z 2 , w  x  y  z , find J  .
  u , v, w 
Solution :
Since u , v, w are exp liciltly given, in terms of x , y , z , we first evaluate
  x, y , z 
J 
  u , v, w 
u u u
x y z
yz zx xy
v v v
We knowthat J    2x 2 y 2z
x y z
1 1 1
w w w
x y z
 yz  2 y  2 z   zx  2 x  2 z   xy  2 x  2 y 
 2  yz  y  z   zx  x  z   xy  x  y  

A
 2  x 2 y  x 2 z  xy 2  xz 2  y 2 z  yz 2 

 2  x 2  y  z   x  y 2  z 2   yz  y  z  

 2  y  z   x 2  x  y  z   yz 
SH
YU
 2  y  z   y  z  x   x  z  x  
 2  y  z  z  x  y  x 
TH

 2  x  y  y  z  z  x 
By u sin g , JJ   1, we get ,
A

  x, y , z 
PR

J 
  u , v, w 
1

2  x  y   y  z  z  x 

152

Get useful study materials from www.rejinpaul.com


www.rejinpaul.com
MA2111/ Engineering Mathematics-I
x2 x3 xx xx
17. If y1  , y2  3 1 , y3  1 2 .
x1 x2 x3
Showthat the Jacobian of y1 , y2 , y3 with respect to x1 , x2 , x3 is 4.
Pr oof :
x2 x3 xx xx
Given : y1  , y2  3 1 , y3  1 2 .
x1 x2 x3
y1 y1 y1
x1 x2 x3
  y1 , y2 , y3  y2 y2 y2
We knowthat 
  x1 , x2 , x3  x1 x2 x3
y3 y3 y3
x1 x2 x3
x2 x3 x3 x2

x12 x1 x1

A
x3 x3 x1 x1
 

SH
x2 x22 x2
x2 x1 x1 x2

x3 x3 x32
YU
 x2 x3 x3 x1 x1 x2
1
 2 2 2 x2 x3  x3 x1 x1 x2
TH

x1 x2 x3
x2 x3 x3 x1  x1 x2
1 1 1
x2 x2 x2
A

 12 22 32 1 1 1
x1 x2 x3
1
PR

1 1
  11  1  1 1  1  1  1
 022  4
Hence the proof .

18. If and . Evaluate without actual substitution.

Solution: and

153

Get useful study materials from www.rejinpaul.com


www.rejinpaul.com
MA2111/ Engineering Mathematics-I

19. If . Find the Jacobian .

Solution:

A
SH
YU
A TH
PR

154

Get useful study materials from www.rejinpaul.com


www.rejinpaul.com
MA2111/ Engineering Mathematics-I
 ( x, y )
23. If x  r cos  , y  r sin  , find
 (r , )
Solution
Given x  r cos  y  r sin 
x y
 cos   sin 
r r
x y
  r sin   r cos 
 
x x
 ( x, y ) r 
We knowthat 
 (r , ) y y
r 
cos  r sin 

sin  r cos 
 r cos 2   r sin 2 

A
 
 r cos 2   sin 2   r

SH
YU
24.If , show that they are not independent. And
also find the relation between them.
TH

Sol: will not be independent if


A
PR

Hence, = = 0.

Hence are not independent.

To find the relation between them:

Given

and

Now,

155

Get useful study materials from www.rejinpaul.com


www.rejinpaul.com
MA2111/ Engineering Mathematics-I

, which is the required relation.

Taylor`s Series and Laurent Series

The Taylor’s series expansion of in the power of and is

Problems:

A
1. Find the Taylor’s series expansion of in the power of x and y upto third degree terms.
Sol: The Taylor’s series expansion of
SH
in the power of and is
YU
A TH
PR

+….…..(1)

, = =1

= =

= , =1

156

Get useful study materials from www.rejinpaul.com


www.rejinpaul.com
MA2111/ Engineering Mathematics-I
,

=0

=2

A
SH
YU
+
TH

= ……..
A
PR

2. Using Taylors series expansion express in powers of x and y upto second

degree terms at

Solution:

157

Get useful study materials from www.rejinpaul.com


www.rejinpaul.com
MA2111/ Engineering Mathematics-I

Taylors series is

3. Expand as Taylors series up to second degree terms

Solution:

A
SH
YU
A TH

Taylors series is
PR

158

Get useful study materials from www.rejinpaul.com


www.rejinpaul.com
MA2111/ Engineering Mathematics-I
16. Use Taylor ' s series oftwo var iables to exp and x y  3 y  2 in powers of 2

x  1 and y  2.
Solution
We knowthat the exp ansion of f ( x, y ) in powers x  a and y  b is given by
f ( x, y )  f ( a , b )  ( x  a ) f x ( a , b )  ( y  b ) f y ( a , b )
1
 ( x  a ) 2 f xx (a, b)  2( x  a )( y  b) f xy (a, b)  ( y  b) 2 f yy (a, b) 
2!
1
 [( x  a )3 f xxx  3( x  a ) 2 ( y  b) f xyy  3( x  a )( y  b) 2 f xyy
3!
 ( y  b)3 f yyy ] ....(1)
Here a  1, b  2
Now f ( x, y )  x 2  3 y  2 f (1, 2)  6
f x  2 xy f x (1, 2)  4
fy  x  3
2
f y (1, 2)  4

A
f xx  2 y f xx (1, 2)  4
f xy  2 x
f yy  0
f xy (1, 2)  2
f yy (1, 2)  0 SH
YU
f xxx  0 f xxx (1, 2)  0
f xxy  2 f xxy (1, 2)  2
f xyy  0 f xyy (1, 2)  0
TH

f yyy  0 f yyy (1, 2)  0


Substituting these value s in (1) we get ,
A

1
x 2 y  3 y  2  6  ( x  1)( 4)  ( y  2)(4)  [( x  1) 2 (4)  2( x  1)( y  2)( 2)  ( y  2) 2 (0)]
PR

2!
1
 [0  3( x  1) 2 ( y  2)(2)  0  0]
3!
 6  4( x  1)  4( y  2)  2( x  1) 2  2( x  1)( y  2)  2( x  1) 2 ( y  2)
Hence the solution.

159

Get useful study materials from www.rejinpaul.com


www.rejinpaul.com
MA2111/ Engineering Mathematics-I
y
15. Expand tan 1 in the neighbourhood of (1,1).
x
solution
y
Let f ( x, y )  tan 1
x
1  y
f x ( x, y )  . 
y 2  x 2 
1 2
x
y 1
 2 , f x (1,1)  
x  y2 2
1 1
f y ( x, y )  .
y2 x
1 2
x
x 1
 2 , f y (1,1) 
x  y2 2

A
f xx ( x, y )   y (1)( x 2  y 2 ) 2 .2 x
2 xy 1

SH
 , f xx (1,1) 
x  y 
2 2 2 2

 x  y 1  x 2 x
2 2
YU
f xy ( x, y ) 
x  y  2 2 2

y 2  x2
 f xy (1,1)  0
TH

x  y2 
2 2

f yy ( x, y )  x(1)( x 2  y 2 ) 2 .2 y
A

2 xy 1
 f yy (1,1)  
x 
PR

2 2
2
y 2
y
tan 1  f ( x, y )
x
1
 f (1,1)  ( x  1) f x (1,1)  ( y  1) f y (1,1)   ( x  1) 2 f xx (1,1)  2( x  1)( y  1) f xy (1,1)  ( y  1) 2 f yy (1,1)   ...
2! 
 u sin g cor : 2 
  1 1 1  1  1 
  ( x  1)     ( y  1)   ( x  1) 2 .  2( x  1)( y  1)0  ( y  1) 2      .....
4  2 2  2!  2  2 
y  1 1 1 1
tan 1   ( x  1)  ( y  1)  ( x  1) 2  ( y  1) 2  ...
x 4 2 2 4 4
Hence the solution

160

Get useful study materials from www.rejinpaul.com


www.rejinpaul.com
MA2111/ Engineering Mathematics-I

Maxima and Minima and Lagrangian Multiplier

Defn: Maximum Value

Defn: Minimum Value

A
SH
YU
Defn: Extremum Value
A TH

Defn: Lagrangian Multiplier


PR

Suppose we require to find the maximum and minimum values of where x,y,z are subject to
a constraint equation

We define a function where

Which is independent of x,y,z

Problems: Examine for its extreme values

Solution: Given

161

Get useful study materials from www.rejinpaul.com


www.rejinpaul.com
MA2111/ Engineering Mathematics-I

At maximum point and minimum point

The points may be maximum points or minimum points.

At and r = 12 > 0

is a minimum point

Minimum value =

At

A
The points are saddle points.

At and r = -12 < 0


SH
YU
is a maximum point
TH

maximum value = .

8. In a plane triangle ABC ,find the maximum value of .


A
PR

Solution:

At the maximum point and minimum point

162

Get useful study materials from www.rejinpaul.com


www.rejinpaul.com
MA2111/ Engineering Mathematics-I
Solving these equations

A+B+C = π

At ,

A
SH
and r < 0
YU
The point is a maximum point.
TH

Maximum value = .

9. Find the volume of the largest rectangular parallelepiped that can be inscribed in the ellipsoid
A
PR

Solution: The given ellipsoid is (1)

The volume of the parallelepiped is (2)

At the max point or min point

(3)

(4)

163

Get useful study materials from www.rejinpaul.com


www.rejinpaul.com
MA2111/ Engineering Mathematics-I
(5)

Solve the equation

(3)x+ (4)y+(5)z 

Put in (3)

Similarly,

Put in (2) Max volume = .

A
SH
10. Find the dimensions of the rectangular box without a top of maximum capacity, whose surface is
108 sq. cm
YU
Solution: Given Surface area
TH

(1)
A

The volume is (2)


PR

At the max point or min point

(3)

(4)

(5)

Solve the equation

(3)x - (4)y

164

Get useful study materials from www.rejinpaul.com


www.rejinpaul.com
MA2111/ Engineering Mathematics-I

(3)x - (5)z

Put in (1)

y =6, z =3

The dimension of the box, having max capacity is Length=6cm, Breadth = 6cm, Height = 3cm.

11. The temperature T at any point (x, y, z) in space is T = 400xy . Find the highest temperature on the

surface of the unit sphere

A
Solution:
SH
(1)
YU
(2)

At the max point or min point


TH

(3)
A
PR

(4)

(5)

Solve the equation

(3)x+ (4)y+(5)z

1600

Put in (3) and (4) we get


165

Get useful study materials from www.rejinpaul.com


www.rejinpaul.com
MA2111/ Engineering Mathematics-I

The highest temperature is = 50.

2 z 2 z 2 z 2 z
13. Pr ovethat    , where x  u cos   v sin  , y  u sin   v cos 
x 2 y 2 u 2 v 2
(OR)
By changing independent var iables u and v to x and y by means of the relations x  u cos   v sin  ,
2 z 2 z 2 z 2 z
y  u sin   v cos  , showthat  transforms in t o  .
u 2 v 2 x 2 y 2

A
SH
YU
A TH
PR

166

Get useful study materials from www.rejinpaul.com


www.rejinpaul.com
MA2111/ Engineering Mathematics-I
Solution :
Here z is a composite function of u and v.
z z x z y
   
u x u y u
z z
 cos   sin 
x y
    
or  z    cos   sin   z
x  x y 
  
  cos   sin   (1)
u x y
z z x z y
Also,    
v x v y v
z z
  sin   cos 
x y
    

A
or  z     sin   cos   z
v  x y 



v

  sin   cos 
x

y
 (2) SH
YU
Now we shall make use of the equivalance of operators as given by (1) and (2).
 2 z   z 
  
TH

u 2 u  u 
    z z 
  cos   sin    cos   sin   (u sin g (1) and (2) )
 u y   x y 
A

2 z 2 z 2 z 2 z
 cos 2  2  cos  sin   sin  cos   sin 2  2
PR

x xy yx y
2 z 2 z 2 z 2 z
ie.,  cos 2
  2 cos  sin   sin 2
  (3)
u 2 x 2 xy y 2
 2 z   z 
Similarly   
v 2 v  v 
    z z 
   sin   cos     sin   cos   (u sin g (3) and (4) )
 x y   x y 

167

Get useful study materials from www.rejinpaul.com


www.rejinpaul.com
MA2111/ Engineering Mathematics-I
 z
2
 z2
 z 2
2 z
 sin 2   sin  cos  cos sin   cos 2

x 2 xy yx y 2
2 z 2 z 2 z 2 z
 sin 2
  2 cos sin   cos 2
  (4)
v 2 x 2 xy y 2
Adding (3) and (4),
2 z 2 z 2 z 2 z
  
u 2 v 2 x 2 y 2
Hence the proof .

1 1
17. Investigatethe max ima of the functions f  x, y   x 2  xy  y 2   .
x y
Solution :
1 1
Given f  x, y   x 2  xy  y 2  
x y

A
f 1
 2x  y  2
x

SH
x
f 1
 x  2y  2
y y
YU
2 f 2
 2 3
x
TH

2
x
2 f 2
 2 3
y 2
y
A

2 f
1
PR

xy
Step1: For a max imum or min imum, we must have
f 1
 0  2x  y  0 i.e., 2 x 3  x 2 y  1  0 ...(1)
x x2
f 1
 0  x  2y  0 i.e., 2 y 3  xy 2  1  0 ...(2)
y y2
(1)  y  2 x3 y  x 2 y 2  y  0 ...(3)
(2)  x  2 xy 3  x 2 y 2  x  0 ...(4)
From (3) we get x y  y  2 x y
2 2 3
...(5)
Substituting (5) in (4) we get

168

Get useful study materials from www.rejinpaul.com


www.rejinpaul.com
MA2111/ Engineering Mathematics-I
2 xy  y  2 x y  x  0
3 3

i.e., 2 x 3 y  2 xy 3  y  x  0
i.e., 2 xy ( x 2  y 2 )  ( x  y )  0
i.e., 2 xy ( x  y )( x  y )  ( x  y )  0
( x  y ) 2 xy ( x  y )  1  0
x  y (or )2 xy ( x  y )  1  0

when x  y, the equation 2 x 3  x 3 y  1  0


gives 2 x 3  x 3  1  0
i.e.,3 x 3  1
1
x3 
3
1 1
x3 y 3
3 3

A
 1 1

SH
Hence  3 , 3  is a critical po int
 3 3
YU
 1 1
Step 2 : At  3 , 3  ,
 3 3
TH

2 f 2
 2 8
x 2
1
A

3
PR

2 f 2 f
 8 ; 1
y 2 xy
2
2 f 2 f  2 f 
.    8 8 1  0
x 2 y 2  xy 
2 f
and 2  8  0
x

 1 1
f ( x, y ) has a min imum at the po int  3 , 3  and
 3 3
4
3
the min imum valueis 3 .

169

Get useful study materials from www.rejinpaul.com


www.rejinpaul.com
MA2111/ Engineering Mathematics-I
18. Find the min imum value of the function
f ( x, y )  x 2  y 2  xy  ax  by.
Solution
Given f ( x, y )  x 2  y 2  xy  ax  by ...(1)
f
 2x  y  a ...(2)
x
f
 2y  x  b ...(3)
y
2 f
2 ...(4)
x 2
2 f
2 ...(5)
y 2
2 f
1 ...(6)
xy

A
Step 1: For miimum value of the function
f
x
 0  2x  y  a ...(7) SH
YU
f
0  2y  x  b ...(8)
y
A TH
PR

170

Get useful study materials from www.rejinpaul.com


www.rejinpaul.com
MA2111/ Engineering Mathematics-I
Solving (7) and (8) we get
2 x  y  a
x  2 y  b
i.e., 4 x  2 y  2a
x  2 y  b

3x  b  2a
b  2a
x
3
b  2a
Substituting x  in (8) we get
3
 b  2a 
  b
y  
3
2

A
(4b  2a )

 a  2b
2
SH
YU
 b  2a 
The critical po int is  , a  2b 
 3 
TH

 b  2a 
Step 2 : At  , a  2b 
 3 
A

2 f 2 f 2 f
 2 ,  2 and 1
x 2 y 2 xy
PR

2
2 f 2 f  2 f 
.    2  2 1
x 2 y 2  xy 
30
 b  2a 
We have min imum value of the function at  , a  2b 
 3 

171

Get useful study materials from www.rejinpaul.com


www.rejinpaul.com
MA2111/ Engineering Mathematics-I
Step 3 :The min imum value of f ( x, y) is obtained by putting
b  2a
x and y  a  2b in f ( x, y ).
3
 b  2a   b  2a   b  2a 
2

   a  2b      a  2b   a    b  a  2b 
2
Minimum value  
 3   3   3 
 b  2 a   b  2a  
     a  2b  a    a  2b   a  2b  b 
 3   3  

 b  2a   4a  5b 
    (a  2b)(a  b0
 3  3 
4ab  5b 2  8a 2  10ab
  a 2  ab  2ab  2b 2
9
4ab  5b  8a 2  9a 2  27ab  18b 2
2

A
9
13b  a  23ab
2 2

SH

9
YU
19. Find the shortest dis tan ce between the lines
x2 y 6 z 5 x5 y 3 z  4
  and  
3 2 2 2 1 6
TH

Solution
x 2 y 6 z 5
Let    ...(1)
2 2
A

3
x 5 y 3 z  4

PR

  ...(2)
2 1 6
Any po int on the first line is P (3  2, 2  6, 2  5) and
any po int on the sec ond line is Q (2   5,   3, 6   4)
PQ  (2  3  5  2  ) 2  (6  2  3   ) 2  (5  2  4  6  ) 2
i.e., PQ 2 17 2  41 2  32  66  114   99
Let f ( ,  )  17 2  41 2  32  66  114   99
f
 34  32   66

f
 32  82   114

2 f 2 f 2 f
 34,  82,  32
 2  2 

172

Get useful study materials from www.rejinpaul.com


www.rejinpaul.com
MA2111/ Engineering Mathematics-I
For a max imum or a min imum of ' f ' we should have
f f
 0, 0
 
i.e., 34  32   66  0
 32  82   114  0

Solving thesetwo equations we get


  1,   1
2 f
At (1, 1)  34  0

2
2 f 2 f  2 f 
.   0
 2  2   
At (1, 1) the function f ( ,  ) has min imum.

A
i.e., At (1, 1) , PQ 2 has min imum which gives the shortest length.

SH
At (1, 1), PQ 2  17  41  32  66  114  99
9
Shortest length  PQ  9  3
YU
A TH
PR

173

Get useful study materials from www.rejinpaul.com


www.rejinpaul.com
MA2111/ Engineering Mathematics-I
1 1 1
20. Find the min imum value of x 2  y 2  z 2 subject to the condition    1
x y z
Solution
Let f  x 2  y 2  z 2
1 1 1
g    1
x y z
Let the auxillary function ' F ' be
1 1 1 
F ( x, y , z )  ( x 2  y 2  z 2 )       1  ...(1)
x y z 
By Lagranges method the values of x, y , z for which ' f ' is min imum are obtained
by the following eqations
F  
 0  2 x  2  0   x3 ...(2)
x x 2
F  
 0  2 y  2  0   y3 ...(3)
y

A
y 2
F  

SH
 0  2z  2  0   z3 ...(4)
z z 2
F 1 1 1
 0    1  0 ...(5)
YU
 x y z
From (2) , (3) and (4) we get

TH

x3  y 3  z 3 
2
1
  3
x y  z  
A

i.e., ...(6)
2
PR

Substituting (6) in (5) we get


3
 1 or x  3
x
3
 1 or y  3
y
3
 1 or z  3
z
(3,3,3) is the po int where min imum value occur. The min imum value of
x 2  y 2  z 2 is 32  32  32  27

174

Get useful study materials from www.rejinpaul.com


www.rejinpaul.com
MA2111/ Engineering Mathematics-I
21. Athin closed rec tan gular box is to have one edge equal to twice the other , and a cons tan t
valume 72m3 . Find theleast surface area of thebox.
Solution
Let the sides of the rec tan gular box be 2 x, x, y.
Then volume is 2 x.x. y  72
i.e., 2 x 2 y  72
i.e., x 2 y  36 ...(1)
The surface area is given by
S  2(2 x.x)  2(2 x. y )  2( x. y )
 4 x 2  6 xy ...(2)
Now we have to find the min imum surface area
S  4 x 2  6 xy under the condition
x 2 y  36

A
Let f  4 x 2  6 xy,
g  x 2 y  36 SH
YU
Let the auxillary function ' F ' be F  f   g
  
F ( x, y )  4 x 2  6 xy   x 2 y  36  ...(3)
TH

By Lagranges method the values of x, y, z for which ' F ' is min mum is obtained
from the following equations
F
 0  8 x  6 y  2 xy  0
A

...(4)
x
PR

F
0 6x   x2  0 ...(5)
y
F
0 00
z
F
0 x 2 y  36  0  x 2 y  36 ...(6)

6
From (5) we get x   ...(7)

6
Substituting x in (6) we get

36
y 2
 2 ...(8)
x

175

Get useful study materials from www.rejinpaul.com


www.rejinpaul.com
MA2111/ Engineering Mathematics-I
6
Substituting x , y   2 in (4) we get

48
  6 2  12 2  0   3   8

i.e.,   2
Substituting   2 in (7) and (8) we get
x  3, y  4.
S has a min imum value at (3, 4)
The min imum value of S  4(3) 2  6(3)(4)
 108

22. Find the max imum value of x m y n z p when x  y  z  a


Solution
Let f  x m y n z p and g  x  y  z  a
Let the auxillary function F  f   g

A
i.e., F ( x, y, z )  x m y n z p    x  y  z  a 

SH
...(1)
F
 0  mx m 1 y n z p    0 ...(2)
x
YU
F
 0  ny n 1 x m z p    0 ...(3)
y
TH

F
 0  p x m y n z p 1    0 ...(4)
z
F
0  x y za 0 ...(5)
A


PR

From (2) , (3) and (4) , we get


   mx m 1 y n z p
   ny n 1 x m z p
   p x m y n z p 1
i.e., mx m 1 y n z p  ny n 1 x m z p  p x m y n z p 1
m n p
i.e.,  
x y z
mn p mn p
 
x yz a

176

Get useful study materials from www.rejinpaul.com


www.rejinpaul.com
MA2111/ Engineering Mathematics-I
Hence max imum value of f occurs when
am
x
mn p
an
y
mn p
ap
z
mn p
The max imum value of
mn nn p p
f  a mn p
m  n  p
mn p

A
SH
YU
A TH
PR

177

Get useful study materials from www.rejinpaul.com


www.rejinpaul.com
MA2111/ Engineering Mathematics-I
UNIT – V Multiple Integrals

Part – A

1. Evaluate

Solution: Let I =

A
2. Evaluate:

Solution: Let I =
SH
YU
A TH

3. Evaluate:
PR

Solution:

4. Evaluate:

Solution: Let I =

178

Get useful study materials from www.rejinpaul.com


www.rejinpaul.com
MA2111/ Engineering Mathematics-I

5. Evaluate:

Solution: Let I =

A
SH
YU
A TH

6. Evaluate:
PR

Solution: I=

7. Evaluate:

Solution: I=

179

Get useful study materials from www.rejinpaul.com


www.rejinpaul.com
MA2111/ Engineering Mathematics-I

8. Evaluate:

Solution: I =

A
9. Find the limits of SH
integration in the double integral
YU

Solution: The limits are: y varies from 0 to 1 and x varies from 0 to 1-y.
TH

10. Change the order of integration


Solution: The given region of integration is bounded by y=0, y=a, x=y & x=a.
A

After changing the order, we have, I =


PR

11. Change the order of integration for the double integral

Ans:

Part – B
1. Evaluate: over the area between y = x2 and y = x.
Solution: The limits are: x varies from 0 to 1 and y varies from x2 to x.
I=

180

Get useful study materials from www.rejinpaul.com


www.rejinpaul.com
MA2111/ Engineering Mathematics-I

2. Evaluate: over the cardioids r = a (1+cosθ).


Solution: The limits of r: 0 to a (1+cosθ) and The limits of θ: 0 to π.
I= =

Put 1+cosθ = t then –sinθ dθ = dt


When θ = 0, t = 2
When θ = π, t = 0.

A
SH
YU
3. Evaluate

Ans: are the polar coordinates for the above integral


A TH
PR

4. Evaluate: over the region in the first quadrant of the circle x2+y2=1.
Solution: In the given region, y varies from 0 to and x varies from 0 to 1.

I=

dx

Put x = sinθ. Then dx = cosθdθ.  varies from 0 to π/2.

181

Get useful study materials from www.rejinpaul.com


www.rejinpaul.com
MA2111/ Engineering Mathematics-I

5. Change the order of integration in I = and hence evaluate it.


Solution: I =

The given region of integration is bounded by x=0, x=1, y=x2 and x+y=2.

In the given integration x is fixed and y is varying.

So, after changing the order we have to keep y fixed and x should vary.

After changing the order we’ve two regions R1 & R2

A
I = I1+I2

I= SH
YU
TH

=
A
PR

6. Evaluate by changing the order of integration.

Solution: The given region is bounded by x=0, x=1, y=x and x2+y2=2.
I=

After changing the order we’ve,


The region R is splinted into two regions R1& R2.
In R1: limits of x: 0 to y & limits of y: 0 to 1
In R2: limits of x: 0 to & limits of y: 1 to
I = I1+I2

182

Get useful study materials from www.rejinpaul.com


www.rejinpaul.com
MA2111/ Engineering Mathematics-I
I1 =

I2 =

A
I=

SH
YU
7. Evaluate by changing the order of integration in
TH

Solution: I =

The given region of integration is bounded by x=0, x=4, y = , y2 = 4x


A
PR

After changing the order we’ve


Limits of x: y2/4 to 2√y
Limits of y: 0 to 4
I= = 16/3.

8. Find the area enclosed by the ellipse

Solution: Area of the ellipse = 4 x area of the first quadrant =4

183

Get useful study materials from www.rejinpaul.com


www.rejinpaul.com
MA2111/ Engineering Mathematics-I
2 2
9. Find the smaller area bounded by y = 2-x and x +y =4.
Solution: Required area =

10. Find the area of the region outside the inner circle r=2cos and inside the outer circle r=4 cos
by double integration.
Solution: Required Area =
=2

11. Find the area of the circle of radius ‘a’ by double integration.
Solution: Transforming Cartesian in Polar coordinates

A
(i.e) x=rcosθ & y=rsinθ. Then dxdy = rdrdθ
limits of θ: 0 to and limits of r: o to
Required Area = 2xupper area SH
YU
=2

=
A TH
PR

12. Find over the area bounded between the circles r = 2sinθ & r= 4sinθ.
Solution:In the region of integration, r varies from r=2sinθ& r= 4sinθ and θ varies from0to π.
I=

13. Find the volume bounded by the cylinder x2+y2=4 and the planes y+z=4 and z=0.
Solution: The limits are:
Z varies from: 0 to 4-y

184

Get useful study materials from www.rejinpaul.com


www.rejinpaul.com
MA2111/ Engineering Mathematics-I
X varies from: - to
Y varies from: -2 to 2.
Required volume = 2 =2

=2 dy= 2

=2
=8 since y is an odd function.

= 16 =16

= 16x2x = 16π
14. Find the volume of the tetrahedron bounded by the plane and
the coordinate plane.
Solution: The limits are:

A
X varies from 0 to a

SH
Y varies from 0 to b

Z varies from o to c
YU
Required Volume =

= =c
TH

=c =c
A

= =
PR

= =

15. Find the volume of the sphere x2+y2+z2=a2 using triple integral.
Solution: Required Volume = 8 x volume in the positive octant = 8
Limits of integration are:
Z varies from 0 to
Y varies from 0 to
X varies from 0 to a
Volume = 8 =8

=8

185

Get useful study materials from www.rejinpaul.com


www.rejinpaul.com
MA2111/ Engineering Mathematics-I
=8

=8 = 2π = 2π

= 2π = cu.units

16. Find the Volume of the ellipsoid


Solution: Required Volume = 8 x Volume in the first octant
Limits of Integration are:

Z varies from 0 to c

Y varies from 0 to b

A
X varies from 0 to a.

Volume = 8 =8
SH
YU
= 8c
TH

= where

= =
A

=2πbc =2πbc =2πbc =


PR

log a x x  y

  e
x y  z
16. Evaluate dzdydx
0 0 0

log a x x  y log a x log a x

  e   [e   (e
x y  z x y  z x y 2( x y )
Solution : dzdydx= ]
0 dydx=  e x  y )dydx
0 0 0 0 0 0 0

 1 4 x 
x
 e 2( x y )   e2x
log a log a
2x 
=    e x  y  dx =   e  e  
  e x  dx
0  2 0 0  2   2 

log a
 e4x 3 2x   e4x 3 2x 
log a

= 
0
  e  e x dx = 
 2 2   2
 e  ex 
2 0

186

Get useful study materials from www.rejinpaul.com


www.rejinpaul.com
MA2111/ Engineering Mathematics-I
1 3  1 3 
=  e 4 log a  e 2 log a  e log a      1
8 4  8 4 

1 4 3 2 3
= a  a a
8 4 8

a a
x 2 dxdy
17. (By changing) express 
0 y x2  y2

Solution:The limits for x are x=y to x=a and the limits for y are y=0 to y=a change to polar co-ordinate

we have x= rcos, y = rsin dxdy = rdrd

a
x = a = rcos  r   a sec 
cos 

The limits are r=0 to r= asec and  = o to /4

A
 / 4 a sec   / 4 a sec 
r 2 cos2  .r.drd r 3 cos2 drd

SH
a a
x 2 dxdy

0 y x2  y2
=  
0 0 r 2 cos2   r 2 sin 2 
=  
0 0
r
YU
 / 4 a sec   /4 a sec   /4
 r3  a3
  r cos drd =    cos2 d =  sec  . cos2 d
3 2 3
=
0 0 0  3 0 3
TH

 /4  /4
a3 1 a3 a3
0 sec  . sec2  d = 3 0 sec  d = 3 [log(sec  tan )]0
3  /4
=
A

3
PR

a3   a3
= [log(sec  tan )  log(sec  tan ] = [log( 2  1)  0]
3 4 4 3

a3
= log( 2  1)
3

18. Find the area inside the circle r=asin but lying outside the cardiod r=a(1-cos)

Solution :

Given curves are r=asin and r =a(1-cos)

The curves intersect where a sin  = a (1-cos)

 a sin  = a –a cos  a sin  + a cos = a  sin  + cos =1

187

Get useful study materials from www.rejinpaul.com


www.rejinpaul.com
MA2111/ Engineering Mathematics-I
1 1 1   1
 sin   cos   sin  cos  cos cos 
2 2 2 4 4 2

 1    
 sin(  )  sin    (or ) 
4 2 4 4 4 4

  2  
   0(or )          
4 4 4 2 2


   0(or ) 
2

 /4 a sin   /2 a sin   /2
 r2 
 sin   (1  cos2   2 cos d
a2
  rdrd     d 2
The required area = = =

0 a (1 cos ) 0  2  a (1cos ) 2 0

 /2  /2
a2  
   
a2

A
sin   cos   1  2 cos d =   1  cos   cos   1  2 cos d 
2 2 2 2
=
2 2  0 

SH
0

 /2  /2

 2 cos  2 cos  d  cos   cos  d


a2 2 a2 2
= = .2
YU
2 0
2 0

  /2
   / 2  1  cos2  
= a 2 sin  0   cos2 d  = a 2 1   
 /2
d 
TH

   0   
0
2

 /2
 1 sin 2    1  
A

= a 1   
2
 = a 2 1    0   0
 2  2  0   2 2  
PR

   a (4   )
2
= a 2 1   
 4 4

17. Find by double integration, the area enclosed by the curves

Ans:

Sub (1) in (2) we get

188

Get useful study materials from www.rejinpaul.com


www.rejinpaul.com
MA2111/ Engineering Mathematics-I

Therefore the point of intersection of (1)&(2) is (0,0) and (4a,4a)

x Varies from 0 to 4a and y varies from

The required Area =

 x 
a b c
19. Evaluate 2
 y 2  z 2 dxdydz

A
0 0 0

a b
 x3 
Solution : I      y 2 x  z 2 x  dydz
0 0 
3 0
c
 c3
a b
SH 
=    cy 2  cz 2 dydz
0 0 
YU
3

b
a
 c 3 y cy 3  a
 c 3b cb3 
=    cyz2  dz =    cbz2  dz
TH

0  0 0 
3 3 3 3

a
A

 c 3bz cb3 z cbz3  c 3ba cb3 a cba3 abc 2


=    =   = [c  b 2  a 2 ]
 3 3 3 0 3 3 3 3
PR

1 2 x
20. Change the order of integration in I    f ( x, y)dydx
0 x2

1 2 x
Solution :Given I    f ( x, y)dydx
0 x2

The given region of integration is bounded by x=0, x=1, y=x2 and x+y=2

In the given integration x is fixed and y is varying

So, after changing the order we have to keep y fixed and x should vary.

After changing the order we have two regions R1 & R2

189

Get useful study materials from www.rejinpaul.com


www.rejinpaul.com
MA2111/ Engineering Mathematics-I
I = I1 + I2

1 y 2 2 x
I   f ( x, y )dxdy    f ( x, y)dxdy
0 0 1 1

1 x 
21.Evaluate :    ( x 2 y  xy 2 )dydx
0x 

1  x  1  x 2  1 x
 x 2 y 2 xy 3 
I=    ( x 2 y  xy 2 )dydx = 0  x
  = 0  2  3  dx
2
Solution : Let ( x y xy ) dydx
0x    x

1
1
 x 3 x.xy3 / 2   x 4 x 4   x4 x7 / 2 5  x5  
=         dx =     
0 
2 3   2 3   8 ( 7 )(3) 6  5 
 2 0

A
1 2 1  21 16  28  9 3
=      (0) 

SH
= =
 8 21 6   168  168 56
YU
21. By Transforming into polar coordinates , Evaluate over annular region between the
circles
TH

Ans: Putting
A
PR

190

Get useful study materials from www.rejinpaul.com


www.rejinpaul.com
MA2111/ Engineering Mathematics-I

A
22. Find the value of
SH
through the positive spherical octant for which
YU
Ans:
A TH
PR

191

Get useful study materials from www.rejinpaul.com

Você também pode gostar